You are on page 1of 74

Daksh Quant Question Bank "June Edition" By Shantanu Shukla

1 www.bankersadda.com | www.sscadda.com | www.careerpower.in | Adda247 App


Daksh Quant Question Bank "June Edition" By Shantanu Shukla
Daksh Quant Question Bank "June Edition" By Shantanu Shukla
Directions (1-15): What approximate value should come Q7. 13.9×6.01÷ 41.89 = ? ÷ 5.9
in place of question mark (?) in the following questions? (a) 15
(b) 12
Q1. (c) 25
(a) 2 (d) 7
(b) 3 (e) 18
(c) 4
Q8. 899÷ 44.8×4.05×69.8 = ?
(d) 5
(a) 6300
(e) 6
(b) 5000
(c) 5600
Q2. ? = 729.11 ÷ 26.97 × 81.11 ÷ 35.91 × 108.18 (d) 5800
(a) 2187 (e) 6000
(b) 6561
(c) 729 Q9. 44.44÷ 4.4÷ 10.1 = (?)²÷ 100
(d) 243 (a) 10
(e) 81 (b) 14
(c) 20
Q3. (d) 15
(a) 3 (e) 25
(b) 12
(c) 8 Q10. 4.9×11.9+8.9 = ? -3.9-11.1
(d) 4 (a) 87
(b) 91
(e) 6
(c) 77
(d) 74
Q4. 47.87% of 350 + 60.11% of 280 = 96.98% of 299.78 +
(e) 84
?% of 150
(a) 30 Q11.
(b) 40 (a) 5
(c) 50 (b) 8
(d) 80 (c) 9
(e) 45 (d) 2
(e) 3
Q5.
(a) –4 Q12.
(b) 6 (a) 26
(c) 8 (b) 33
(d) 5 (c) 23
(e) 4 (d) 20
(e) 40
Q6. 59.77% of 880+79.9% of 591 = ?
Q13. 1.101+ 11.01 + 101.01 ÷ 1.01 = ?
(a) 1000
(a) 109
(b) 950
(b) 116
(c) 1100 (c) 101
(d) 1050 (d) 113
(e) 900 (e) 117

2 www.bankersadda.com | www.sscadda.com | www.careerpower.in | Adda247 App


Daksh Quant Question Bank "June Edition" By Shantanu Shukla
Q18. If in E INC got 15000 more valid votes than AAP and
Q14.
BJP won the election by 15000 votes, then find the valid
(a) 10
(b) 12 votes received by BJP. (There are only three parties in E –
(c) 14 BJP, INC & AAP)
(d) 20 (a) 40000
(e) 15 (b) 38000
(c) 45000
(d) 35000
Q15.
(e) 30000
(a) 324
(b) 413
Q19. In B there are only two parties – BJP & INC. If BJP got
(c) 400
60% of total votes in B and ratio of invalid votes received
(d) 343
by BJP and INC is 2 : 1, then find valid votes received by
(e) 363
INC in B.
Directions (16-20): Study the bar chart given below and (a) 24000
answer the following questions. (b) 27000
Bar chart shows the total votes (in ‘000) in 5 different (c) 22000
cities (A, B, C, D & E) and percentage of valid votes out of (d) 20000
total votes in these 5 cities. (e) 25000

Q20. In C there are four parties – BJP, INC, SP & AAP. If


ratio of valid votes received by BJP, INC, SP & AAP in C is 4
: 2 : 3 : 3 and ratio of invalid votes received by BJP, INC, SP
& AAP in C is 1 : 3 : 4 : 2, then find difference between
total votes received by INC & SP in C.
(a) 2000
(b) 3500
(c) 3400
(d) 2100
(e) 2700
Q16. In A, ratio of valid votes received by BJP, INC & SP is
11 : 3 : 5. If BJP win the election by 24000 votes, then find Directions (21-25): Given bar graph shows the details of
valid votes received by INC & SP together. number of students in a particular class of 3 different
(a) 28000 schools in 5 different years.
(b) 40000
(c) 45000
(d) 32000
(e) 21000

Q17. If in D there are only two parties-BJP & INC and BJP
got 70% of the total valid votes, then find by how many
votes BJP won the election in D.
(a) 18000
(b) 25000
(c) 10000
(d) 16000
(e) 6000

3 www.bankersadda.com | www.sscadda.com | www.careerpower.in | Adda247 App


Daksh Quant Question Bank "June Edition" By Shantanu Shukla
Q21. What is the difference between average number of Directions (26-30): Given bar graph shows the
students of school A across all the years and the average percentage distribution of total number of students of
number of students of school B across all the years? each school (P, Q, R & S) who took admission in 3 different
(a) 18 streams. Total students in P, Q, R & S are 700, 800, 400 &
(b) 10 900 respectively.
(c) 12
(d) 14
(e) 16

Q22. Find the respective ratio of the total number of


students of school A in 2011 and 2012 together to the
total number of students of school C in 2013 and 2014
together?
(a) 31:33
(b) 47:55
(c) 55:47
(d) 33:31 Q26. What is average number of students who have opted
(e) 31:37 for MBBS in all the 4 colleges?
(a) 256
Q23. If in 2016, the total number of students in School A, (b) 233
(c) 284
School B and School C increases by 10%,20% and 15%
(d) 224
respectively as compared to 2015, then find the total
(e) 296
number of students in 2016 in all the schools together?
(a) 850
Q27. What is the ratio of the total number of student who
(b) 870
have opted for both engg. and MBBS stream together in
(c) 780 college Q to that of in same stream together in college R?
(d) 830 (a) 38:65
(e) 800 (b) 67:35
(c) 35:67
Q24. Total students of all the school together in 2013 is (d) 65:38
approximately what percentage more/less than the total (e) 29:37
students of school B in 2011 and 2015 together?
(a) 52% Q28. The number of student who have opted for MBBS in
(b) 59% college P is what percent of the number of students who
(c) 56% have opted for the engg. in college Q?
(d) 63% (a) 87.5%
(e) 48% (b) 50%
(c) 75%
Q25. Find the difference between the number of total (d) 100%
students from all the schools in 2011 and 2013 together (e) 62.5%
and the total number of students from all the schools in
2014 and 2015 together? Q29. What is the ratio of the no. of students who have
(a) 140 opted for engg. in college R to that of those who have
opted for same stream in college P?
(b) 60
(a) 14:11
(c) 120
(b) 17:13
(d) 80
(c) 11:14
(e) 100
(d) 13:17
(e) None of these

4 www.bankersadda.com | www.sscadda.com | www.careerpower.in | Adda247 App


Daksh Quant Question Bank "June Edition" By Shantanu Shukla
Q30. Which of the combination represents the colleges Q32. Find the respective ratio of Creta car sold in Delhi
with maximum number of students, who have opted for and Mohali together to the total of Innova car sold in
pharmacy and those who have opted for engg. Kolkata and Ahmedabad together?
respectively? (a) 41:35
(a) P & R (b) 46:53
(b) Q & S (c) 26:35
(c) Q & R (d) 35:41
(d) R & S (e) 35:54
(e) P & Q
Q33. Find the total number of cars sold in Kolkata?
Directions (31-35): following line graph shows the data (a) 1140
of 3 different types of cars sold in 5 different cities. (b) 1170
(c) 1250
(d) 1300
(e) 1080

Q34. Find the difference between number of Honda city


cars sold in delhi and number of creta cars sold in surat?
(a) 70
(b) 110
(c) 80
(d) 100
(e) 90

Q35. Find the average number of Honda city car sold in all
Q31. Number of Honda city car sold in Ahmedabad is the cities?
what percent of total Innova car sold in Surat? (a) 420
(a) (b) 426
(c) 416
(b) (d) 430
(e) 435
(c)

(d) Directions (36-40): Following Line Graph shows the


marks scored by Student A and Student B in high school in
(e)
different Subjects.(Maximum Marks is 100 for each
subject). Study the data carefully and answer the
following questions.

5 www.bankersadda.com | www.sscadda.com | www.careerpower.in | Adda247 App


Daksh Quant Question Bank "June Edition" By Shantanu Shukla
Q36. What is difference between average marks scored by
Student A and Student B in all subjects?
(a) 1.75
(b) 1.45
(c) 1.50
(d) 1.25
(e) 1

Q37. What is Ratio of marks obtained by Student A in


Maths and Computer together to the marks obtained by
Student B in Science and English together?
(a) 7:5
(b) 7:8
(c) 8:7
(d) 8:5
Note – Buses manufactured by a company in any year =
(e) 5:7
(Sold + unsold) buses of that company in that year.
Q38. What is the overall percentage marks scored by
Student B? Q41. If company – B sold 80%, 90% and 80% buses
(a) 68.75 % manufactured by it in 2016, 2017 & 2018 respectively,
(b) 67.5 % then find average number of unsold buses of company – B
(c) 68% in 2016, 2017 & 2018.
(d) 67% (a) 400
(e) 69.25% (b) 900
(c) 500
Q39. Marks Scored by Student A in Math is what percent (d)200
of marks scored by Student B in Science and English (e) 100
together?
(a) 40% Q42. Buses manufactured by company – A in 2016 &
(b) 60% 2018 together are what percent more or less than buses
(c) 50% manufactured by company – D in 2017 & 2018 together?
(d) 70%
(a) 50%
(e) 80%
(b) 90%
(c) 70%
Q40. If passing marks for each subject is 40% of 120, then
(d) 60%
what is the difference between passing marks and marks
(e) 80%
scored by Student B in Computer?
(a) 30
(b) 32 Q43. If buses sold by company – B and company – E in
(c) 36 2016 are 75% and 80% respectively, then find ratio of
(d) 40 buses sold by company – B & E together in 2016 to unsold
(e) 45 buses of company – B & E together in 2016.
(a) 11 : 5
Directions (41-45): Study the radar chart given below (b) 5 :1
and answer the following questions. (c) 8 :5
Radar chart shows the number of buses manufactured by (d) 7 : 2
5 different companies (A, B, C, D& E) in 2016, 2017 & (e) None of the above.
2018.

6 www.bankersadda.com | www.sscadda.com | www.careerpower.in | Adda247 App


Daksh Quant Question Bank "June Edition" By Shantanu Shukla
Q44. Buses manufactured in 2018 by all these 5 Q50. 10, 20, 60, 300, ?, 23100
companies together are approximately what percent of (a) 1650
buses manufactured in 2016 by all these 5 companies (b) 1500
together? (c) 1800
(a) 104% (d) 2100
(b) 108% (e) 2400
(c) 102%
(d) 118% Q51. 3, 8, 18, 33, 53, ?
(e) 115% (a) 72
(b) 80
Q45. Average number of buses manufactured by company (c) 76
– B, C & D in 2017 are how much more or less than buses (d) 78
manufactured by company – D & E together in 2016? (e) 73
(a) 1500
(b) 2500 Q52. 9, 64, 25, 216, ?, 512
(c) 2000 (a) 49
(d)1000 (b) 343
(e) 500 (c) 81
(d) 100
Directions (46-50): Find the missing term in the (e) 121
following number series questions.
Q53. 12, 36, 80, 164, 328, ?
Q46. 6, 7, 16, 51, 208, ?
(a) 648
(a) 970
(b) 664
(b) 845
(c) 660
(c) 1085
(d) 656
(d) 985
(e) 652
(e) 1045

Q54. 15, 23, 30, 36, 41, ?


Q47. 2000, ?, 2164, 2308, 2504, 2760
(a) 48
(a) 2049
(b) 52
(b) 2036
(c) 49
(c) 2064
(d) 45
(d) 2100
(e) 2081 (e) 51

Q48. 800, 770, 728, 672, ?, 510, Q55. 7, 14, 28, ?, 112, 224
(a) 616 (a) 56
(b) 600 (b) 64
(c) 580 (c) 58
(d) 624 (d) 62
(e) 560 (e) 60

Q49. 500, 548, 620, ?, 836, 980 Q56. 250, 375, 591, ?, 1446, 2175
(a) 716 (a) 954
(b) 736 (b) 934
(c) 756 (c) 914
(d) 696 (d) 894
(e) 746 (e) 974

7 www.bankersadda.com | www.sscadda.com | www.careerpower.in | Adda247 App


Daksh Quant Question Bank "June Edition" By Shantanu Shukla
Q57. 30, 90 , 360 , 1800 , 10800, ? Q63. 65% of 180 + ?% of 210 = 80% of 225
(a) 54000 (a) 45
(b) 73200 (b) 30
(c) 72800 (c) 40
(d) 75600 (d) 50
(e) 64800 (e) 25

Q58. 39600 , 6600 , ?, 330, 110, 55 Q64.


(a) 1320 (a) 145
(b) 1650 (b) 115
(c) 1100 (c) 120
(d) 1160 (d) 135
(e) 1280 (e) 125

Q59. 200, ?, 236 , 284, 380 , 572 Q65.


(a) 228 (a) 8
(b) 208 (b) 12
(c) 224 (c) 7
(d) 220 (d) 6
(e) 212 (e) 4

Q60. 8000, 7100, 6475 , 6075 , ?, 5750 Q66.


(a) 5975
(a)
(b) 5850
(c) 5675 (b)
(d) 5875
(e) 5775 (c)

Directions (61-70): What should come in place of (d)


question mark (?) in the following questions?
(e)

Q61. 1528 + 525 ÷ 25 – 840 = 510 + ?


Q67.
(a) 199
(a) 17
(b) 299
(b) 23
(c) 159 (c) 15
(d) 189 (d) 13
(e) 165 (e) 11

Q62. Q68.
(a) 259 (a) 1.75
(b) 169 (b) 1.20
(c) 157 (c) 1.25
(d) 129 (d) 1.45
(e) 141 (e) 1.80

8 www.bankersadda.com | www.sscadda.com | www.careerpower.in | Adda247 App


Daksh Quant Question Bank "June Edition" By Shantanu Shukla
(a) Rs. 95
Q69.
(b) Rs. 228
(a) 19
(c) Rs. 285
(b) 37
(d) Rs. 380
(c) 13
(d) 23 (e) Rs. 114
(e) 17
Q75. Average weight of A, B and C is 93 kg.. If another
Q70. man D joins the group whose weight is 81 kg then new
(a) 23.5 average of the four people will be equal to: -
(b) 13 (a) 65 kg
(c) 21.5 (b) 67 kg
(d)17.5 (c) 86 kg
(e) 19.5 (d) 90 kg
(e) 96 kg
Q71. In an alloy A, Aluminum and Nickel are present in
the ratio 4 : 3 and in alloy B, the same element are in the Q76. An article when sold at 4/5 of its original selling
ratio 3 : 5. If these two alloys be mixed to form a new alloy
price, gives a profit of 20%. Find the profit % when the
in which same elements are in the ratio 1 : 1, then find the
same article is sold at its actual selling price.
ratio of alloy A and alloy B in the new alloy ?
(a) 15 %
(a) 6 : 7
(b) 20 %
(b) 7 : 4
(c) 4 : 7 (c) 25%
(d) 7 : 6 (d) 22 %
(e) 4 : 3 (e) None of these

Q72. The average wt. of boys in school is 60kg while Q77. An amount of Rs. 20000 when invested at R% simple
average wt. of girls is 55 kg. The average wt. of both boys interest for 2 years becomes Rs. 24000. What will it
and girls is 58kg. Find the number girls in school if become in 3 years if invested at (R+2)%? (in Rs.)
number of boys is 720. (a) 27200
(a) 480 (b) 26300
(b) 720 (c) 25200
(c) 240 (d) 27400
(d) 360 (e) 28100
(e) 600
Q78. Karan purchased an article marked up by 50% at a
Q73. If 21 is added in a number, the result becomes
discount of 20% but later he found that the article was
116⅔% of itself. Find the new number?
having a defect so he decided to return it but the
(a) 126
shopkeeper returned him only 90% of what he had paid.
(b) 147
(c) 130 What is profit/loss (in %)of the shopkeeper in the whole
(d) 136 transaction?
(e) 125 (a) 8
(b) 10
Q74. The amount invested by P and Q is in the ratio 2 : 3 (c) 12
and that invested by P and R is 5 : 7. If the profit earned by (d) 15
P at the end of year is Rs. 76 less than that earned by R. (e) None of these
Find the profit earned by Q.

9 www.bankersadda.com | www.sscadda.com | www.careerpower.in | Adda247 App


Daksh Quant Question Bank "June Edition" By Shantanu Shukla
Q79. A sum of Rs. x was invested at 10% simple interest Q84. The interest earned on an amount after 2 yrs at 10
for 3 years. If the same sum was invested at 4% more for % per annum compounded yearly is Rs 672. Find the
same period, then it would have fetched Rs. 120 more. interest earned on same amount after 4 yr at 14 % per
Find the value of 5x. (in Rs.) annum at simple interest ?
(a) 5000 (a) Rs 1792
(b) 4800 (b) Rs 1864
(c) 3600 (c) Rs 1912
(d) 5500
(d) Rs 1754
(e) 4000
(e) Rs 1720
Q80. A sum of Rs. P was invested at 10% for 2 years at
simple interest. If the same sum was invested at 20% for Q85. If the shopkeeper marked the price of an item 60%
‘x’ years, it would have fetched Rs. 200 more. Find ‘x’ if Px above the cost price and then gives two successive
= 5000. (value of x is given in months) discount of 10% and 15% respectively, then find the
(a) 12 profit percentage of the shopkeeper on selling the item?
(b) 18 (a) 25.4%
(c) 15 (b) 22.4%
(d) Cannot be determined (c) 20%
(e) None of these (d) 28.5%
(e) 32%
Q81. A sum of Rs 1400 becomes Rs 2408 in 8 yrs at
simple interest, then find the rate of interest for last 4
Q86. Johny calculates his profit at cost price while Jini at
yrs,if the interest rate for 1st 4 yrs is 12% per annum ?
selling price. If cost price is same for all and everyone
(a) 8 %
(b) 10 % calculate their profit as 10%. Find ratio of selling price.
(c) 6% (a) 100:111
(d) 4 % (b) 10:11
(e) None of these (c) 10:101
(d) 99:100
Q82. Cost price of 2 bags is in ratio 4:5 and these bags are (e) Cannot be determined
sold at 10% profit & 20% profit respectively. Find overall
profit percentage in entire transaction. Q87. An amount doubles in 5 years at simple interest. In
what time will it become 12 times of itself at same rate?
(a)
(in years)
(b) (a) 30
(b) 50
(c) (c) 55
(d) 36
(d) (e) None of these
(e)
Q88. Aakash sells an article at a profit of 10%.Had he
Q83. Rs. 12000 becomes Rs. 15000 in 18 months at a bought it for 5% less and sold it for 120 rs more then he
certain rate of interest at simple interest. Find amount if would have gained 20% profit. What is the cost price of
Rs. 5000 invested at same rate for 30 months at simple the article ?
interest. (a)Rs 2500
(a) Rs. 7883.33 (b) Rs 4000
(b) Rs. 7083.33
(c) Rs 3000
(c) Rs. 7279.80
(d) Rs 3500
(d) Rs. 7173.33
(e) Rs 2000
(e) None of these

10 www.bankersadda.com | www.sscadda.com | www.careerpower.in | Adda247 App


Daksh Quant Question Bank "June Edition" By Shantanu Shukla
Q89. An amount of 4000 rs is invested at 20% per annum Q95. 1656, 549, 180, ?, 16
for 2 yrs at compound interest compounding half-yearly, (a) 63
then find the total interest amount received after 2 yrs ? (b) 73
(a) Rs 1856.4 (c) 85
(b) Rs 1812.4 (d) 57
(c) Rs 1882.4 (e) 67
(d) Rs 1912.4
(e) None of these Directions (96-101): Line graph shows the quantity of 5
different products purchased by a person.
Q90. Anu bought a purse at a discount of 20% which was
marked at 30% higher than cost price. A customer gets
10% extra discount on purchase of more than two purse.
If Anu purchased 3 such purses, find profit or loss percent
of shopkeeper.
(a) 4% loss
(b) 4% profit
(c) 6.4% profit
(d) 6.4% loss
(e) None of these

Directions (91-95): What will come in the place of the


question mark (?) in the following number series?
Q96. If sum of per kg cost of sugar and that of salt is Rs.90
and the ratio between per kg cost of sugar and that of salt
Q91. 5, 7, 25, 131, ?, 8335
is 3 : 2. Then, find the difference of total cost of sugar and
(a) 845
(b) 940 total cost of salt ?
(c) 965 (a) Rs. 530
(d) 925 (b) Rs. 630
(e) 825 (c) Rs. 670
(d) Rs. 750
Q92. 81, 86, 94, 111, 135, ? (e) Rs. 720
(a) 172
(b) 176 Q97. If total cost of Tea is Rs. 5000 and that of wheat is
(c) 192 Rs. 450. Then cost per kg of wheat is what percent more
(d) 182 or less than cost per kg of Tea ?
(e) 186 (a) 72%
(b) 86%
Q93. 61, 32, 55, 36, 53, ? (c) 82%
(a) 50 (d) 78%
(b) 42 (e) 92%
(c) 40
(d) 65 Q98. One kg of rice and one kg of sugar is purchased in Rs
(e) 48 450. If cost per kg of rice decreases by 33 ⅓% & cost per
kg of Sugar increases by 33 ⅓% then total cost per kg of
Q94. 5, ?, 11.5, 35, 164, 1360 rice and sugar is Rs. 500. Then find cost per kg of sugar ?
(a) 7.5 (a) Rs. 300
(b) 11 (b) Rs. 350
(c) 8.5 (c) Rs. 200
(d) 9 (d) Rs. 250
(e) 5.5 (e) Rs. 450

11 www.bankersadda.com | www.sscadda.com | www.careerpower.in | Adda247 App


Daksh Quant Question Bank "June Edition" By Shantanu Shukla
Q99. If cost per kg of Tea & per kg of Rice is Rs. 220 & Rs. Q103. Arun sells his watch at a profit of 33 ⅓% & his
50 respectively then find the ratio of total cost of tea to purse at a loss of 16 ⅔% & on whole he gains Rs. 50. And
total cost of rice ? if he sells his watch at a loss of 16 ⅔% & purse at profit of
(a) 53 : 15 33 ⅓% then there will be no profit no loss. Find cost price
(b) 44 : 17 of the watch ?
(c) 41 : 17 (a) Rs. 300
(d) 47 : 15 (b) Rs. 100
(e) 44 : 15 (c) Rs. 250
(d) Rs. 200
Q100. Total quantity of sugar and salt purchased together (e) Rs. 150
is what percent more/less than the total quantity of Tea &
wheat purchased together ? Q104. A boat can cover an equal distance in upstream and
in downstream in 6 hours. If speed of boat in still water is
(a)
200% more than the speed of stream then find the time
taken to cover the same distance in upstream.
(b)
(a) 5 hours
(c) (b) 3 hours
(c) 4.5 hours
(d) (d) 3.5 hours
(e) 4 hours
(e)
Q105. Prabhat invested Rs. 15600 on SI at rate of R% p.a.
Q101. If cost per kg of sugar, salt & rice is Rs. 10, Rs. 30 & for 3 years & the interest obtained is Rs. 7020. If he
Rs. 20 respectively then find the sum of difference of total invested the same amount at rate of (R+5)% p.a. for two
cost of sugar and that of salt and difference of total cost of years on CI then find the interest obtained by Prabhat?
sugar and that of rice?
(a) Rs. 6864
(a) Rs. 500
(b) Rs. 6250
(b) Rs. 475
(c) Rs. 6748
(c) Rs. 400
(d) Rs. 6468
(d) Rs. 450
(e) Rs. 6648
(e) Rs. 435

Q102. A train crosses a tunnel which is half of its length


with a speed of 144 km/hr. in ½ min, then find the time in
which it will cross another train which is double of its
length and standing on platform in opposite direction
with 60% of its initial speed ?
(a) 120 sec.
(b) 90 sec.
(c) 150 sec.
(d) 100 sec.
(e) 180 sec.

12 www.bankersadda.com | www.sscadda.com | www.careerpower.in | Adda247 App


Daksh Quant Question Bank "June Edition" By Shantanu Shukla
Directions (106-110): Given table shows the data of Q110. Which sections have equal number of students?
students of a class related to results of Half-yearly and (a) section A & B
Annual examination. Study the data carefully and answer (b) section A & C
the questions. (c) section B & C
(d) all have same no.of students
(e) none

Directions (111-115): Following table gives the detail of


items sold by two different stores i.e Store A and Store B
and among them percentage of numbers of items
purchased by females are given.

Q106. How many students are there in Section B of class?


(a) 50
(b) 60
(c) 90
(d) 100
(e) 110
Q111. Items purchased by females from store A on
Q107. Students passed in both exams in all sections are Wednesday and Thursday together is how much percent
what percent more/less than students failed in both more/less than the total items purchased by males from
exams in all sections? store B on Thursday and Friday together ?

(a)
(a)
(b)
(b)

(c) (c)

(d) (d)
(e)
(e)
Q112. Find the respective ratio between total number of
Q108. what is average of students who passed in only one items purchased by males from store A on Tuesday and
examination in all sections together? Wednesday together to the total numbers of items
(a) 39.67 purchased by females from store B on Thursday and
(b) 40.67 Friday together ?
(c) 41.67 (a) 45:73
(d) 42.67 (b) 41:71
(e) 43.67 (c) 73:41
(d) 71:41
(e) 37:71
Q109. Students failed in both exams in section C are what
percent of total students in section C? (in %)
Q113. Find the total number of items purchased by males
(a) 30
from store B on all the given days together?
(b) 20
(a) 936
(c) 18 (b)832
(d) 25 (c) 912
(e) 33.33 (d) 852
(e) 882

13 www.bankersadda.com | www.sscadda.com | www.careerpower.in | Adda247 App


Daksh Quant Question Bank "June Edition" By Shantanu Shukla
Q114. Total Items purchased on Thursday and Friday Q118. Find the respective ratio of the total number of
together of store A is what percentage of total items magazines distributed among the distributors of company
purchased on Wednesday and Thursday together of store R to that of the total no. of magazines distributed among
B ?
distributors of company T?
(a) 125%
(b) 100% (a) 21:19
(c) 120% (b)19:21
(d) 140% (c) 17:21
(e) 80% (d) 21:17
(e) 17:23
Q115. If total items purchased from store A and Store B
on Saturday are 20% more and 30% more respectively
Q119. Find the average number of books distributed
than the total items sold by store A and B on Wednesday,
then find the total number of items purchased from Store among the distributors by all the five companies together?
A and Store B together on saturday? (a) 2784
(a) 828 (b) 2664
(b) 753 (c) 2680
(c) 783 (d) 2756
(d) 807
(e) 2724
(e) 823

Directions (116-120): Following table DI gives the detail Q120. Find the difference between total no. of
of magazines printed by five different companies and distributors of magazines sold by companies P and Q
distributed among different distributors and answer the together to the total no. of distributors of magazines sold
following question accordingly. by companies R and T together?
(a) 38
(b) 34
(c) 36
(d) 42
(e) 40

Note:- magazines were equally distributed among the Directions (121-135): What will come in the place of the
distributors of respective printing companies. question mark (?) in the following number series?

Q116. What is the average no. of magazines distributed Q121. 2, 6, 42, 142, 338, ?
by companies Q, R and T among their respective
(a) 665
distributors?
(a) 2720 (b) 632
(b) 2640 (c) 631
(c) 2480 (d) 682
(d) 2960 (e) 662
(e) 3120
Q122. 87, 89, 86, 91, 84, ?
Q117. Find the total numbers of distributors of magazines
of company Q and T together? (a) 95
(a) 62 (b) 96
(b) 72 (c) 97
(c) 84 (d) 98
(d) 78 (e) 90
(e) 64

14 www.bankersadda.com | www.sscadda.com | www.careerpower.in | Adda247 App


Daksh Quant Question Bank "June Edition" By Shantanu Shukla
Q123. 77, 81, 72, 88, 63, ? Q130. 15, 30, 50, 80, 130, ?
(a) 98 (a) 200
(b) 99 (b) 210
(c) 101 (c) 220
(d) 105 (d) 230
(e) 97 (e) 240

Q131. 4, 18, 48, ?, 180, 294


Q124. 3120, 624, 156, ?, 26
(a) 96
(a) 84
(b) 120
(b) 100
(c) 100
(c) 102 (d) 90
(d) 52 (e) 115
(e) 150
Q132. 2, 4, 12, 60, ?, 4620
Q125. 7, 21, 105, 735, ? (a) 300
(a) 6235 (b) 380
(b) 3256 (c) 480
(c) 6755 (d) 660
(d) 6515 (e) 420
(e) 6615
Q133. 2, 3, 5, 8, 13, ?
Q126. 1511, 302, 75, 24, ?, 5 (a) 21
(b) 19
(a) 12
(c) 17
(b) 10
(d) 24
(c) 8
(e) 26
(d) 15
(e) 9 Q134. 150, 395, ?, 900, 1160, 1425
(a) 650
Q127. 7, 10, 18, 42, 90, ? (b) 625
(a) 150 (c) 676
(b) 160 (d) 645
(c) 170 (e) 680
(d) 210
(e) 240 Q135. 100, 96, 87, 71, ?, 10
(a) 35
Q128. 48, 24, 72, 18, 90, ? (b) 21
(a) 9 (c) 46
(d) 52
(b) 12
(e) 25
(c) 21
(d) 18
Directions (136-140): What will come in place of the
(e) 15 question mark (?) in the following number series?

Q129. 9, 6, 9, 24, 108, ? Q136. 16, 20, 28, 44, ?


(a) 888 (a) 82
(b) 864 (b) 76
(c) 872 (c) 60
(d) 878 (d) 52
(e) 882 (e) 96

15 www.bankersadda.com | www.sscadda.com | www.careerpower.in | Adda247 App


Daksh Quant Question Bank "June Edition" By Shantanu Shukla
Q137. 2, 11, 36, 85, ?, 287 Q142. If total selling price of Quant book for store A and B
(a) 163 together is Rs. 510. Find M.R.P. of Quant for store B?
(b) 166 (a) Rs.240
(c) 170 (b) Rs.270
(d) 185 (c) Rs.280
(e) 206 (d) Rs.300
(e) Rs.600
Q138. 8, 6, 14, 40, ?
(a) 151 Q143. If market price of a reasoning book was 50% more
(b) 148 than cost price of the book for store C. Find profit percent
(c) 80 on selling a reasoning book by store C?
(d) 162 (a) 20%
(e) 98 (b) 15%
(c) 25%
Q139. 3, 83, 152, 208,249 , ? (d) 10%
(a) 280 (e) 12.5%
(b) 320
(c) 265 Q144. What is the ratio of average discount given on
(d) 351 quant book by store A, reasoning book by store C and
(e) 273 English book by store B to market price of a book?
(a) 5:4
Q140. 2, 14, 70, ?, 420 (b) 2:3
(a) 190 (c) 3:2
(b) 320 (d) 1:5
(c) 210 (e) 4:5
(d) 200
(e) 315 Q145. If an English book is sold at Rs. 170 by store A, the
find selling price of reasoning book for store C?
Directions (141-145): Study the paragraph carefully and (a) Rs. 160
answer the following questions. (b) Rs. 170
Adda247 publications sold three books i.e. Quant, (c) Rs. 135
English and reasoning on three different stores i.e. A, B (d) Rs. 105
and C. (e) None of these.
Quant, reasoning and English book are sold at 20%
discount by store A, C and B respectively. Quant, Directions (146-150): Pie chart given below shows no.
reasoning and English book are sold at 15% discount by of non-defective article manufactured by five different
store C, B and A respectively. Discount percent given on firms i.e. P, Q, R, S and T. Read the data carefully and
Quant book by store B is half of discount percent given on answer the following questions. (total article
reasoning book by store C. M.R.P. for each book is same at manufactured by any firm = defective + non-defective
every store. article)

Q141. Store A sold reasoning book at Rs. 880, find M.R.P


of the book if discount given by store A on reasoning book
is 20% more than discount given by store B on quant
book?
(a) Rs. 1200
(b) Rs. 1000
(c) Rs. 960
(d)Rs. 1240
(e) None of these.

16 www.bankersadda.com | www.sscadda.com | www.careerpower.in | Adda247 App


Daksh Quant Question Bank "June Edition" By Shantanu Shukla
Q146. No. of non-defective article manufactured by firm Q Q151. 4 men & 3 children completes a project for Rs. 600
is what percent of non-defective article manufactured by in 3 days. If a man completes same project in 15 days.
firm T? Find daily wage of a man.
(a) 60% (a) Rs 36
(b) 62.5% (b) Rs.40
(c) 37.5% (c) Rs.44
(d) 50% (d) Rs.48
(e) 72.5% (e) Rs.42

Q147. If ratio of non-defective to defective article Q152. Difference between 50% of y and 10% of x is 170
manufactured by firm S is 75:2, then find ratio of non- whereas difference between 40% of x and 30% of y is
defective article manufactured by firm T to defective
zero. Find the sum of ‘x’ and ‘y’ ?
article manufactured by firm S?
(a) 770
(a) 37:1
(b) 630
(b) 34:1
(c) 600
(c) 33:1
(d) 700
(d) 24:1
(e) 560
(e) 38:1

Q148. Average no. of non-defective article manufactured Q153. If ratio of time periods of investment of P and Q is
by firm Q and R is what percent of total no. of non- 4:5, profit at the end of the year is 75000 and P’s share is
defective article manufactured by firm S? Rs 15000, then what is the ratio of Q’s and P’s
(a) 72⅔% investment?
(b) 66⅔% (a) 5:16
(c) 33⅓% (b) 6:7
(d) 73⅓% (c) 12:13
(e) None of these. (d) 16:5
(e) 8:5
Q149. If no. of defective article manufactured by firm P
and R are 30% and 30
10
% respectively of total no. of Q154. Average of 8 consecutive odd numbers is 10. What
13
article manufactured by each of these firm, then find will be the average of smallest 4 numbers out of 8
defective article manufactured by firm P is what percent numbers?
more/ less than that of firm R? (a) 7
(a) 20% (b) 8
(b) 15% (c) 6
(c) 50% (d) 4
(d) 25% (e) 5
(e) 75%
Q155. The work done by 5 boys in 20 days can be done by
Q150. What is the ratio of non-defective article 10 men in 8 days. 4 Men & 4 boys undertook a work to
manufactured by firm Q and R together to that of P and S complete in 3 days for Rs. 540. Find the amount earned by
together? boys for their whole contribution.
(a) 2:1 (a) Rs 236
(b) 1:1 (b) Rs.240
(c) 1:2 (c) Rs.244
(d) 1:3 (d) Rs.248
(e) 3:1 (e) Rs.242

17 www.bankersadda.com | www.sscadda.com | www.careerpower.in | Adda247 App


Daksh Quant Question Bank "June Edition" By Shantanu Shukla
Q156. Sanjay scored 56% marks and passed an exam by Directions (161-165): Given line graph shows the sum
10 marks while Rohit scored 48% marks but failed by 6 invested, rate of interest and time period of investment by
marks. What is the pass percentage? 4 people. Study the data carefully and answer the
(a) 52.5% questions.
(b) 51.5% (NOTE: all invested their sum at simple interest)
(c) 52%
(d) 51%
(e) None of these

Q157. Four books are to be distributed among seven


students. If no students gets more than one book, then the
number of ways possible to do it is?
(a) 180
(b) 240
(c) 260
(d) 210 Q161. How much will Rohit receive after completion of
(e) 220 his investment period? (in Rs.)
(a) 5200
Q158. In a basket, there are 8 red ball and 6 green ball. If (b) 6800
2 balls are taken out from the basket, then find what is the (c) 4800
probability of both ball being either red or green? (d) 4400
(a) 43/91 (e) 4600
(b) 47/91
(c) 51/91 Q162. Interest amount received by Mahesh is what
percent more than interest amount received by Karan?
(d) 43/87
(a) 85%
(e) 43/82
(b) 60%
(c) 75%
Q159. The parallel sides of a trapezium are 4 cm & 10 cm
(d) 70%
respectively while non-parallel sides are equal to side of
(e) 80%
square of area 25 sq.cm. find area of trapezium. (in
sq.cm.) Q163. What is total amount received as interest by
(a) 50 Anurag & Rohit together? (in Rs.)
(b) 42 (a) None of these
(c) 56 (b) 3150
(d) 28 (c) 3200
(e) 14 (d) 3360
(e) 3420
Q160. The ratio of area of square to that of rectangle of
length 10 cm is 4 : 5. If breadth of rectangle is same as Q164. If Karan had invested same sum at compound
side of square. Find length of diagonal of square. interest at same rate of interest for same period. How
(a) 9√2 cm much more would he earn?
(b) 10√2 cm (a) Rs 80
(c) 6√2 cm (b) Rs 90
(d) 4√2 cm (c) Rs 70
(e) 8√2 cm (d) Rs 60
(e) None of these

18 www.bankersadda.com | www.sscadda.com | www.careerpower.in | Adda247 App


Daksh Quant Question Bank "June Edition" By Shantanu Shukla
Q165. Who among the four had received the highest Q169. Find the average marks scored in physics subject
amount as interest? by all the given five students together?
(a) Karan (a) 105
(b) Anurag (b) 110
(c) Both Anurag & Mahesh (c) 108
(d) Rohit (d) 100
(e) Mahesh (e) 98

Directions (166-170): Following Table chart gives the Q170. Total marks scored by Aakash, Siddharth and
details of 5 students of a particular school in five different Lokesh in English is what percentage of the total marks
subjects in the annual exam. scored by Amit, Aakash and lokesh in maths?
(a) 75%
(b) 70%
(c) 65%
(d) 68%
(e) 80%

Directions (171-175): Given pie graph shows percentage


Note:-the data provided in the table is percentage of distribution of watches manufactured by a company in
marks out of total marks in that particular subject. 2018. Study the graph carefully & answer the questions.

Q166. Total marks scored by lokesh in physics, chemistry


and maths together is how much more/less than total
marks scored by Amit in the same three subjects
together?
(a) 75
(b) 65
(c) 69
(d) 55
(e) 80

Q167. Find the overall percentage of marks scored by


Siddharth in the exam?
(a) 75% Q171. What is average of watches manufactured by Casio,
(b) 82% Titan & Sonata together?
(c) 68% (a) 500
(d) 72% (b) 600
(e) 80% (c) 400
(d) 200
Q168. Find the difference of total marks scored by Ritesh (e) 300
in all the given subject together and total marks scored by
Aakash in all the given subjects together? Q172. What is ratio of watches manufactured by Timex &
(a) 71 Sonata together to that by Fossil & Casio together?
(a) 5:4
(b) 84
(b) 8:7
(c) 78
(c) 7:8
(d) 82
(d) 7:4
(e) 93
(e) 5:8

19 www.bankersadda.com | www.sscadda.com | www.careerpower.in | Adda247 App


Daksh Quant Question Bank "June Edition" By Shantanu Shukla
Q173. Watches manufactured of Sonata are what percent Q177. If the ratio of males to females in pharmacy and
more/less than watches manufactured of Rado? finearts departments are 1:2 and 3:2 respectively, then
(a) 130% find the total number of females in pharmacy and finearts
(b) 150% together?
(c) 200% (a) 784
(d) 170% (b) 712
(e) 100% (c) 736
(d)756
Q174. If next year, Titan watch production increases by (e) 812
10% while that of Timex decreases by 10%. What is
difference in manufacturing of both in next year? Q178. Find the central angle of the total students of
(a) 80 architecture department of the university?
(b) 90 (a) 64.8˚
(c) 100 (b) 75.6˚
(d) 65 (c) 72˚
(e) 75 (d) 43.2˚
(e) 68.4˚
Q175. No. of watches manufactured of how many brands
is more than average no. of watches manufactured?
Q179. Number of students who failed in the final
(a) 4
semester exam from MBBS and Finearts dept are 20% and
(b) 3
15% respectively of their respective dept, then find the
(c) 1
total number of student who passed the semester from
(d) 2
(e) 5 MBBS and Finearts dept ?
(a) 1345
Directions (176-180): Study the charts given below (b) 1323
carefully and answer the following questions. (c) 1368
Pie chart shows the distribution of total students of a (d) 1420
university in different departments as shown below. (e) 1456

Q180. Total students from engineering and pharmacy


department together is approximately what percentage of
the total students from MBBS and Fine arts dept?
(a) 122%
(b) 148%
(c) 126%
(d) 143%
(e)134%

Q176. Find the ratio of total number of student from


engineering and architecture department together to total
students from the pharmacy and BSc department
together?
(a) 27:43
(b) 27:47
(c) 43:27
(d) 47:27
(e) 37:42

20 www.bankersadda.com | www.sscadda.com | www.careerpower.in | Adda247 App


Daksh Quant Question Bank "June Edition" By Shantanu Shukla
Directions (181-185): Study the charts given below Q185. Total spectators of cricket and football together is
carefully and answer the following questions. how much more/less than total spectators of badminton
and tennis together?
Pie chart shows the percentage distribution of total (a) 160
Spectators of a particular city loving different sports as (b) 140
shown below. (c) 180
(d) 200
(e) None of these

Directions (186-190): In each of these questions, two


equations (I) and (II) are given. You have to solve both the
equations and answer the following questions.

Q186.

(a) x > y
(b) x < y
(c) x ≥ y
(d) x ≤ y
(e) x = y or no relation.

Q181. Total spectators of Badminton and kabaddi Q187.


together is what percentage of total spectators of cricket
and hockey together ? (a) x > y
(a) 70% (b) x < y
(b) 75% (c) x ≥ y
(c) 80% (d) x ≤ y
(d) 65% (e) x = y or no relation.
(e) 60%
Q188.
Q182. Find the ratio of total spectators of Football and
tennis together to the total spectators of Cricket? (a) x > y
(a) 17:12 (b) x < y
(b) 11:15
(c) x ≥ y
(c) 15:11
(d)12:17 (d) x ≤ y
(e) 13:18 (e) x = y or no relation.

Q183. Find the central angle of total spectators of Q189.


badminton and tennis together?
(a) 79.2˚ (a) x > y
(b) 136.8˚ (b) x < y
(c) 115.2˚ (c) x ≥ y
(d) 126˚ (d) x ≤ y
(e) 133.2˚ (e) x = y or no relation.

Q184. Out of total hockey spectators, male and female Q190.


lovers are in the ratio 9: 6 respectively, then find
2
difference between male and female spectators of hockey?
(a) x > y
(a) 524
(b) 484 (b) x < y
(c) 336 (c) x ≥ y
(d) 504 (d) x ≤ y
(e) 472 (e) x = y or no relation.

21 www.bankersadda.com | www.sscadda.com | www.careerpower.in | Adda247 App


Daksh Quant Question Bank "June Edition" By Shantanu Shukla
Directions (191-195): In each of these questions, two Directions (196-200): In each of these questions, two
equations (I) and (II) are given. You have to solve both the equations (I) and (II) are given. You have to solve both the
equations and give answer equations and give answer

Q191. Q196.

(a) if x > y (a) if x > y


(b) if x ≥ y
(b) if x ≥ y
(c) if x < y
(c) if x < y
(d) if x ≤ y
(d) if x ≤ y
(e) if x = y or no relation can be established between x and
(e) if x = y or no relation can be established between x and
y
y
Q197.
Q192.
(a) if x > y
(a) if x > y (b) if x ≥ y
(b) if x ≥ y (c) if x < y
(c) if x < y (d) if x ≤ y
(d) if x ≤ y (e) if x = y or no relation can be established between x and
(e) if x = y or no relation can be established between x and y
y
Q198.
Q193.
(a) if x > y
(a) if x > y (b) if x ≥ y
(b) if x ≥ y (c) if x < y
(c) if x < y (d) if x ≤ y
(d) if x ≤ y (e) if x = y or no relation can be established between x and
(e) if x = y or no relation can be established between x and y
y
Q199.
Q194.

(a) if x > y
(a) if x > y
(b) if x ≥ y
(b) if x ≥ y
(c) if x < y
(c) if x < y
(d) if x ≤ y (d) if x ≤ y
(e) if x = y or no relation can be established between x and (e) if x = y or no relation can be established between x and
y y

Q195. Q200.

(a) if x > y
(b) if x ≥ y (a) if x > y
(c) if x < y (b) if x ≥ y
(d) if x ≤ y (c) if x < y
(e) if x = y or no relation can be established between x and (d) if x ≤ y
y (e) if x = y or no relation can be established between x and
y

22 www.bankersadda.com | www.sscadda.com | www.careerpower.in | Adda247 App


Daksh Quant Question Bank "June Edition" By Shantanu Shukla
Directions (201-220): What should come in place of Q206. 80% of ? = √250 × 44 + 40% of 8500
question mark (?) in the following questions? (a) 80
(b) 120
Q201. 48% of 525 + ? % Of 250 = 499 (c) 150
(a) 88.8 (d) 180
(b) 76.6 (e) 240
(c) 82.6
(d) 98.8 594 2300
Q207. ? × 40 ÷ 24 × 27 = 115 ×
(e) 92.8 264
(a) 1
5 7 1 (b) 2
Q202. 2 of 8 of 28 of 1600 = 260 + ? – 499
(c) 3
(a) 264 (d) 4
(b) 480 (e) 5
(c) 364
(d) 342
Q208. 20% 𝑜𝑓 (40 × √? ) = (32)2 + (16)²
(e) 420
(a) 160
(b) 2560
Q203. √52 × 41 × 5 – 172 – 75 =?
(c) 16
(a) 69
(d) 25600
(b) 71
(c) 79 (e) 256
(d) 63
(e) 89 Q209. ? + 13× 50 = 420 + 45% of 800 + 220
(a) 300
Q204. √256 × 49 + (19)2 + 11 = (? )2 (b) 350
(a) 484 (c) 400
(b) 22 (d) 450
(c) 24 (e) 250
(d) 42 3
(e) 26 Q210. (? )2 = 256 × (2)8 ÷ (8)5 × 32
(a) 4
Q205. 252 + 520 ÷ 20 + 420 = 121 + ? (b) 256
(a) 587 (c) 64
(b) 577 (d) 1024
(c) 527 (e) 16
(d) 477
(e) 627 4
4 𝑜𝑓25 5 3 1
Q211. ( 5
48
) ÷ (4 𝑜𝑓32 + 7 𝑜𝑓 21) =? 𝑜𝑓 49

(a) 3.5
(b) 3
(c) 2.5
(d) 4
(e) 5
1
Q212. √? 𝑜𝑓 6 + 20% 𝑜𝑓 95 = 𝑜𝑓 62
2
(a) 3
(b) 4
(c) 5
(d) 6
(e) 7

23 www.bankersadda.com | www.sscadda.com | www.careerpower.in | Adda247 App


Daksh Quant Question Bank "June Edition" By Shantanu Shukla
5
Q213. ( 𝑜𝑓 6 𝑜𝑓
3 9 2
) +? = 45 Q220. 2450 +3760 -3830 =6000 - ?
3 5 11 (a) 3610
(a) 5 (b) 3620
(b) 7 (c) 3580
(c) 4 (d) 3600
(d) 8 (e) 3520
(e) 6
Directions (221-240):- What approximate value will
Q214. (7 ×
4 14 3 4
÷ 2) − (10 𝑜𝑓 ? ) = 5 − 3 come in place of question mark (?) in the following
5 questions. (You are not expected to find the exact value)
(a) 10
(b) 8 125.98
Q221. 154.03 ×
198.02 156.05 51.03
− 101.98 × 78.03 = ?
(c) 9 17.99
(a) 8
(d) 11
(b) 25
(e) 12 (c) 35
(d) 50
4 1 27 1
Q215. 4 5 + 2 15 − 5
= 2 5 ÷ 3 ×? (e) 0
2
(a)
9 Q222. 80.08% 𝑜𝑓 349.98 + 45.02% 𝑜𝑓 799.99 = ? % ×
(b) 1 255.95
(c) 2 (a) 300
(d) 3 (b) 270
(e) 9
1 (c) 235
(d) 250
(e) 200
Q216. √5776 - √1444 + √729 =43 + ?
(a) 25 Q223. √1224.99 ÷ 6.99 = ? – 1799.98
(b) 20 (a) 1600
(c) 26 (b) 1810
(d) 24 (c) 1950
(e) 22 (d) 1710
(e) 1900
Q217. 78 ×26÷6 +1262= 1311 + (?)2
(a) 17 Q224. 2744.98 – 1417.99 = ? + 987.98
(a) 369
(b) 22
(b) 299
(c) 15
(c) 119
(d) 13 (d) 229
(e) 19 (e) 339

Q218.1484÷28 + 1462÷34 -12×7= ? Q225. ?2 = 44.99 % of 4500.02– 24.99% of 3959.98 +


(a) 12 87.01 × 2.97
(b) 14 (a) 0
(c) 18 (b) 16
(d) 16 (c) 36
(e) 20 (d) 56
(e) 80
Q219. 42.5×15 +37.5× 25= 1420 + ?
Q226. 1749.98 ÷ 350 × 49.79 + 111.03 = (? )²
(a) 145
(a)19
(b) 165 (b) 39
(c) 155 (c) 29
(d) 170 (d) 9
(e) 185 (e) 49

24 www.bankersadda.com | www.sscadda.com | www.careerpower.in | Adda247 App


Daksh Quant Question Bank "June Edition" By Shantanu Shukla
Q227. ? × 625.04 = 15625.01 + 9999.99 Q235. 1485.988 + 212.04 − 1703.99 = ? −(11.02)²
(a) 41 (a) 95
(b) 25 (b) 115
(c) 60 (c) 130
(d) 12 (d) 102
(e) 68 (e) 135
64.02 1
Q228. 29.98% of 701 – 350.01 + 82% of 501 = ? Q236. 43.495 × 31.99 × 28.979 – 2.012 =?
(a) 230 (a) 4
(b) 290 (b) 12
(c) 270 (c) 6
(d) 250 (d) 1
(e) 310 (e) 8

Q229. 5759.99 ÷ 45.01 + 11.99 = ?× 10.03 Q237. (33.33 × 80.989 ÷ 99.99) + 3.024−? = 4.012
(a) 60 (a) 20
(b) 2 (b) 26
(c) 34
(c) 46
(d) 16
(d) 30
(e) 40
(e) 14
Q238. 20.021 + 4.969 + 30.499 − 50.022 =?
Q230. 1395.98 + 412.04 − 2703.99 = ? −(31.02)² (a) 5.5
(a) 28 (b) 2
(b) 45 (c) 8.5
(c) 65 (d) 12.5
(d) 85 (e) 14
(e) 98
Q239. 995.013 − 39.976 × 19.99 + 5.022 = 1.988 ×?
Q231. 41.979 ×
22
+ 19.989% of 530.014 – 26.021 =? (a) 115
7 (b) 85
(a)244 (c) 100
(b) 198 (d) 125
(c) 236 (e) 75
(d) 212
(e) 252 Q240. (10.011)2 + (23.989)2 = 275.99 +(?)2
(a) 34
Q232. (23.012 × 22.989) + 20.985 × 7.014 = ? ² (b) 6
(a) 8 (c) 28
(b) 38 (d) 12
(c) 26 (e) 20
(d) 12
(e) 44 Directions (241-260): What should come in place of
question mark (?) in the following questions.
Q233. √1443.979 ÷ 18.981 + 3.5 × √16.017 = (? )
(a) 16 Q241. 0.5, 1, 1.5, ?, 0.75, 0
(a) 2
(b) 30
(b) 1.5
(c) 8 (c) 1.25
(d) 26 (d) 1
(e) 10 (e) 0.75
Q234. 779.98 ÷ 48.014 × 15.989 = ? Q242. 5, 15, 45, 135, ?, 1215
(a) 280 (a) 415
(b) 248 (b) 395
(c) 275 (c) 410
(d) 242 (d) 405
(e) 260 (e) 400

25 www.bankersadda.com | www.sscadda.com | www.careerpower.in | Adda247 App


Daksh Quant Question Bank "June Edition" By Shantanu Shukla
Q243. 90, 96, 102, 108, 114, ? Q251. 21, 22, ?, 35, 51, 76
(a) 116 (a) 28
(b) 124 (b) 23
(c) 118 (c) 24
(d) 122 (d) 26
(e) 120 (e) 29

Q244. 389, 380, 370, 359, ?, 334 Q252. 128, ?, 32, 16, 8, 4
(a) 347 (a) 64
(b) 345 (b) 60
(c) 351 (c) 68
(d) 350 (d) 56
(e) 348 (e) 72

Q245. 1, 3, 6, ?, 18, 29 Q253. 16, 22, 28, 34, 40, ?


(a) 10 (a) 44
(b) 11 (b) 46
(c) 9 (c) 48
(d) 12 (d) 42
(e) 8 (e) 50

Q246. 280, 295, 325, 370, 430, ? Q254. 1, 8, 27, ?, 125, 216
(a) 515 (a) 68
(b) 525 (b) 66
(c) 505 (c) 62
(d) 490 (d) 60
(e) 520 (e) 64

Q247. 4, 2, 3, 7.5, ?, 118.125 Q255. 20, ?, 12, 19, 39, 98.5


(a) 24.25 (a) 9
(b) 28.25 (b) 10
(c) 27.25 (c) 11
(d) 25.25 (d) 24
(e) 26.25 (e) 12

Q248. 18, 25, 30, ?, 42, 49 Q256. 31, 33, 36, ?, 48, 59
(a) 37 (a) 38
(b) 35 (b) 37
(c) 39 (c) 43
(d) 41 (d) 41
(e) 43 (e) 40

Q249. 1, 2, 4, 8, ?, 32 Q257. 6, 36, 180, 720, ?, 4320


(a) 32 (a) 3600
(b) 24 (b) 1080
(c) 12 (c) 1440
(d) 16 (d) 2880
(e) 20 (e) 2160

Q250. 121, ?, 169, 196, 225, 256 Q258. 23, 29, ?, 41, 47, 53
(a) 148 (a) 33
(b) 144 (b) 35
(c) 140 (c) 37
(d) 136 (d) 36
(e) 132 (e) 39

26 www.bankersadda.com | www.sscadda.com | www.careerpower.in | Adda247 App


Daksh Quant Question Bank "June Edition" By Shantanu Shukla
Q259. 1, 5, ?, 30, 55, 91 Q266. 7, 4, 5, 8.5, 20, 52.5, 160.5
(a) 13 (a) 4
(b) 10 (b) 20
(c) 9 (c) 8.5
(d) 52.5
(d) 14
(e) 7
(e) 18
Q267. 160, 207, 260, 319, 380, 449, 518
Q260. 5, 10, 20, 35, 55, ? (a) 160
(a) 85 (b) 319
(b) 75 (c) 207
(c) 80 (d) 449
(d) 70 (e) 380
(e) 65
Q268. 12 , 6 , 6, 12, 36, 231, 1848
(a) 12
Directions (261-280): Find the wrong term in the (b) 1848
following number series questions. (c) 36
(d) 231
Q261. 110, 156, 210, 282, 342, 420, 506 (e) Series is right
(a) 342
(b) 282 Q269. 14700, 2100, 12600, 2500, 10080, 3360, 6720
(c) 110 (a) 2100
(b) 12600
(d) 420
(c) 10080
(e) 506 (d) 3360
(e) 2500
Q262. 2000, 2000, 1000, 3000, 600, 3750, 625
(a) 1000 Q270. 20.25, 23.04, 26.01, 29.16, 32.56, 36.00,
(b) 3750 39.69
(c) 625 (a) 36.00
(d) 600 (b) 23.04
(c) 32.56
(e) 3000
(d) 20.25
(e) 29.16
Q263. 2, 2, 5, 17, 72, 359, 2159
(a) 72 Q271. 8, 12, 24, 60, 180, 640, 2520
(b) 359 (a) 60
(c) 5 (b) 180
(d) 17 (c) 2520
(e) 2159 (d) 640
(e) 8
Q264. 9000, 7920, 7020, 6300, 5760, 5400, 5200
(a) 5400
(b) 9000
(c) 6300
(d) 7020
(e) 5200

Q265. 100, 120, 154, 192, 248, 320, 410


(a) 100
(b) 248
(c) 410
(d) 154
(e) 120

27 www.bankersadda.com | www.sscadda.com | www.careerpower.in | Adda247 App


Daksh Quant Question Bank "June Edition" By Shantanu Shukla
Q272. –1, 1, 2, 6, 14, 30, 62 Q280. 2, 6, 11, 23, 47, 95, 191
(a) 1 (a) 6
(b) 62 (b) 11
(c) -1 (c) 47
(d) 14 (d) 2
(e) 30 (e) 23

Q273. 3, 4, 12, 41, 103, 228, 444 Directions (281-300):- In each of the following
(a) 228 questions, two equations (I) and (II) are given. Solve the
(b) 3
equations and mark the correct option:
(c) 41
(a) if x>y
(d) 444
(b) if x≥y
(e) 103
(c) if x<y
Q274. 5, 3, 4, 7, 17, 45, 138 (d) if x ≤y
(a) 3 (e) if x = y or no relation can be established between x and
(b) 7 y.
(c) 17
(d) 45 Q281. I. 𝑥 2 − 21𝑥 + 110 = 0
(e) 138 II. 𝑦 2 − 25𝑦 + 156 = 0

Q275. 17, 25, 38, 53, 79, 107, 140 Q282. I. 𝑥 2 + 29𝑥 + 208 = 0
(a) 79 II. y2 + 35y +306= 0
(b) 140
(c) 25 3
Q283. I. 𝑥 = √4096
(d) 53 II. 𝑦 2 + 121 = 377
(e) 107
Q284. I. 3x2+23x+44=0
Q276. 102, 83, 66, 50, 38, 27, 18
II.4y2+33y+65=0
(a) 102
(b) 83
(c) 38 Q285. I. 𝑥 2 + 41𝑥 + 418 = 0
(d) 50 II. 𝑦 2 + 47𝑦 + 550 = 0
(e) 66
Q286. I. 2x² - 17x + 36 = 0
Q277. 2, 12, 36, 80, 150, 251, 392 II. 3y² - 22y + 40 = 0
(a) 36
(b) 80 Q287. I. x² + 21x + 108 = 0
(c) 251 II. y² + 14y + 48 = 0
(d) 392
(e) 150 Q288. I. 2x² + 7x – 60 = 0
II. 3y² - 28y + 64 = 0
Q278. 2, 3, 5, 7, 11, 15, 17
(a) 3 Q289. I. x² - 2x – 24 = 0
(b) 11 II. y² + 3y – 40 = 0
(c) 15
(d) 17 Q290. I. 4x² + 27x + 45 = 0
(e) 7
II. 5y² + 42y + 88 = 0
Q279. 11, 22, 34, 47, 61, 77, 92
Q291. I. 𝑥 2 + 5𝑥 + 6 = 0
(a) 77
(b) 61 II. 𝑦 2 + 9𝑦 + 14 = 0
(c) 92
(d) 22 Q292. I. 𝑥 2 − 18𝑥 + 45 = 0
(e) 34 II. 𝑦 2 + 12𝑦 − 45 = 0

28 www.bankersadda.com | www.sscadda.com | www.careerpower.in | Adda247 App


Daksh Quant Question Bank "June Edition" By Shantanu Shukla
Q293. I. 9𝑥 2 + 11𝑥 + 2 = 0 Q304.If the shopkeeper marked the price of an item 60%
II. 8𝑦 2 + 6𝑦 + 1 = 0 above the cost price and then gives two successive
discount of 10% and 15% respectively, then find the
Q294. I. 6𝑥² + 5𝑥 + 1 = 0 profit percentage of the shopkeeper on selling the item?
II. 4𝑦² – 15𝑦 = 4 (a) 25.4%
(b) 22.4%
Q295. I. 𝑥 2 + 3𝑥 = 0
(c) 20%
II. 𝑥 2 + 𝑦 = 10
(d) 28.5%
Q296. I. 𝑥 2 − 25𝑥 + 156 = 0 (e) 32%
II. 𝑦 2 − 29𝑦 + 210 = 0
Q305.Aakash sells an article at a profit of 10%.Had he
Q297. I. 𝑥 = 196
2 bought it for 5% less and sold it for 120 rs more then he
II. 𝑦 = √196 would have gained 20% profit. What is the cost price of
the article ?
Q298. I. 𝑥 2 + 12𝑥 + 35 = 0 (a)Rs 2500
II. 𝑦 2 + 14𝑦 + 48 = 0 (b) Rs 4000
(c) Rs 3000
Q299. I. 3𝑥² + 23𝑥 + 30 = 0 (d) Rs 3500
II. 𝑦² + 15𝑦 + 56 =0 (e) Rs 2000

Q300. I. 𝑥 2 + 17𝑥 + 72 = 0
Q306. Kappu & Chandu have their speed in ratio 5:6. If
II. 𝑦 2 + 13𝑦 + 42=0
both start from 2 points 110 kms away towards each
4 other. How much distance Chandu had travelled more
Q301. An article when sold at of its original selling price,
5 than Kappu when they meet for first time? (both start at
gives a profit of 20%. Find the profit % when the same same time)
article is sold at its actual selling price. (a) 11 kms
(a) 15 %
(b) 20 kms
(b) 20 %
(c) 25% (c) 10 kms
(d) 22 % (d) Cannot be determined
(e) None of these (e) None of these

Q302. Akshay buys an article and markup it 30 % above Q307. What will be the time taken by Rahul to cover the
its cost price. At the time of sale, he same distance which is covered by Abhishek in 5 hours if
gives 10% discount instead of 15% due to which he earns ratio of speed of Abhishek and Rahul is 6 : 5?
Rs. 13 more. Find cost price. (a) 4 hrs
(a) Rs. 230 (b) 5 hrs
(b) None of these (c) 6 hrs
(c) Rs. 150
(d) 7 hrs
(d) Rs. 130
(e) Rs. 200 (e) 3 hrs

Q303. Cost price of 2 bags is in ratio 4:5 and these bags Q308. Manoj takes twice the time to cover a distance ‘D’
are sold at 10% profit & 20% profit respectively. Find km than time taken by Shreya to cover 2D km. Manoj
overall profit percentage in entire transaction. started from his home & after 30 min, Shreya started from
5
(a) 15 % his house but she catched him after travelling for 20/3
9
5 km. Find speed of Shreya. (speed is considered in kmph)
(b) 12 9 %
(a) 40 kmph
5
(c) 18 9 % (b) 13.33 kmph
5
(d) 14 9 % (c) 28.5 kmph
7 (d) 17.77 kmph
(e) 12 9 %
(e) None of these

29 www.bankersadda.com | www.sscadda.com | www.careerpower.in | Adda247 App


Daksh Quant Question Bank "June Edition" By Shantanu Shukla
Q309. If train – A starts from P at 8:00 am and train B Q314. In an alloy A, Aluminum and Nickel are present in
starts from Q at 10:00 am towards Q and P respectively. If the ratio 4 : 3 respectively and in alloy B, the same
total distance between P to Q is 594 km and speed of train element are in the ratio 3 : 5 respectively. If these two
A and train B are 63 km/hr and 54 km/hr respectively, alloys be mixed to form a new alloy in which same
then find at what distance from Q will they cross each elements are in the ratio 1 : 1 respectively, then find the
other?(in km) ratio of alloy A and alloy B in the new alloy ?
(a) 6 : 7
(a) 208
(b) 7 : 4
(b) 216
(c) 4 : 7
(c) 192 (d) 7 : 6
(d) 180 (e) 4 : 3
(e) 224
Q315.The ratio of milk and water in a mixture of 64 litres
Q310. Time taken by Dhoni to cover a distance of ‘D’ km is 7:1.How much water must be added to it so that the
is same as time taken by Rohit to cover ‘3D’ km. if Virat is ratio of milk and water becomes 14:5?
50% faster than Rohit and when Dhoni & Virat travel (a) 12 litres
towards each other from points D km apart then they (b) 15 litres
meet after 2 hours. Find time taken by Virat to cover D (c) 8 litres
km. (d) 16 litres
(a) 2 hours (e) None of these
(b) 3.2 hours
(c) 2.44 hours Q316. A person is 16 yrs older than his son. After 2 yrs,
the person’s age will be double the age of his son. Then
(d) 3.67 hours
find the age of his son 8 yrs hence?
(e) 2.68 hours (a) 24 yrs
(b) 20 yrs
Q311. A vessel contains mixture of petrol and diesel (c) 22 yrs
contains 75% petrol. When some quantity of kerosene oil (d) 18 yrs
is added in the vessel then mixture contains 50% petrol. (e) 28 yrs
Find ratio of quantity of kerosene oil to diesel in the final
mixture. Q317. Mahesh has two sons named Karan and Arjun. The
(a)8 : 3 ratio of present age of Mahesh and Karan is 5 : 2 and that
(b)2 : 1 of Karan and Arjun is 4 : 3. Also, Karan is 5 years elder
(c)1 :2 than Arjun. Find the ratio of their ages 10 years ago.
(d)3 :8 (a) 10 ∶ 4 ∶ 3
(e) None of the above. (b) 7 ∶ 2 ∶ 1
(c) 8 ∶ 2 ∶ 1
(d) 8 ∶ 3 ∶ 1
Q312.In a vessel, the ratio of alcohol and water is 3:7. If
(e) 12 ∶ 6 ∶ 5
20 litres of the mixture is taken out and 2 litres of water is
filled into it, the new ratio becomes 1:3. Find the initial Q318. Suman is 25 yrs elder to his son. If 7 yrs hence, the
quantity of the mixture in the vessel? ratio of ages of suman and his son will be 2:1, then how
(a) 40 litre many years back from present suman’s son was born?
(b) 20 litre (a) 20 yrs
(c) 30 litre (b) 24 yrs
(d) 50 litre (c) 15 yrs
(e) None of these (d) 18 yrs
(e) None of these
Q313.Type A milk of cost price Rs 35 per litre is mixed
with Type B milk having cost price rs 50 per litre in the Q319. 5 yrs hence, the age of shivam increased by 20%
ratio 3:2 respectively, then find the selling price(per kg) of and 6 yrs ago the age of Ayush was 25% less than his
the final mixture when sold at 25% profit ?(in rs) present age. What is the sum of the ages of shivam and
Ayush, 8 yrs hence?
(a) 51.25
(a)54 yrs
(b) 48.75
(b)60 yrs
(c) 53.3 (c) 65 yrs
(d) 57.4 (d) 56 yrs
(e) 53.5 (e) 69 yrs

30 www.bankersadda.com | www.sscadda.com | www.careerpower.in | Adda247 App


Daksh Quant Question Bank "June Edition" By Shantanu Shukla
Q320. The present ages of Father and his son is in the Q325. A sum of Rs. x was invested at 10% simple interest
ratio 3:1 respectively and if 6 years later ,the ratio of their for 3 years. If the same sum was invested at 4% more for
ages becomes 7:3, then find the age of the son 3 years same period, then it would have fetched Rs. 120 more.
ago? Find the value of 5x. (in Rs.)
(a) 12 yrs (a) 5000
(b) 9 yrs (b) 4800
(c) 15 yrs (c) 3600
(d) 18 yrs (d) 5500
(e) 14 yrs (e) 4000

Q321. Jaddu & Ravi invested equal amount at 10% p.a. Q326. Arshad & Sanjay undertake a piece of work for Rs
rate of interest on simple interest & compound interest. 600. Arshad can do it in 15 days and Sanjay can do it in 30
Interest received by Ravi after 2 years is same as interest days. With the help of Vidya, they finish it in 5 days. How
received by Jaddu after some years. Find investment much Vidya should be paid for her contribution?
period of Jaddu.
(a) 𝑅𝑠 350
(a) 2.8 years
(b) Rs 400
(b) None of these
(c) Rs 300
(c) 2.4 years
(d) Rs 325
(d) 2.1 years
(e) Rs 380
(e) Cannot be determined

Q322.Shivam invested Rs 30000 at a rate of interest 20% Q327. 4 men & 3 children complete a project for Rs. 600
p.a. The interest was compounded half-yearly for first in 3 days. If a man completes same project in 15 days.
year and in the next year it was compounded yearly. What Find daily wage of a man.
will be total interest earned at the end of the 2 yrs ? (a) Rs 36
(a) Rs 12960 (b) Rs. 40
(b) Rs 14800 (c) Rs. 44
(c) Rs 15600 (d) Rs. 48
(d) Rs 13560 (e) Rs. 42
(e) Rs 13980
Q328. The work done by 5 boys in 20 days can be done by
Q323. A man invested Rs.P in three different schemes – A, 10 men in 8 days. 4 Men & 4 boys undertook a work to
B and C in the ratio 2 : 1 : 3 respectively. He invested in complete in 3 days for Rs. 540. Find the amount earned by
scheme A at the rate of 10% p.a. at SI for 2 years, in boys for their whole contribution.
scheme B at the rate of 5% p.a at C.I compounded (a) Rs 236
annually for 2 years and in scheme C at the rate of 6% p.a (b) Rs. 240
at CI compounded half yearly for 1 year and received total (c) Rs. 244
interest of Rs. 6852. Find the value of P. (d) Rs. 248
(a) Rs.60000 (e) Rs. 242
(b) Rs.72000
(c) Rs.48000
(d) Rs.120000
(e) can’t be determined.

Q324.Aakash invested Rs 16800 on simple interest at r %


p.a. for 3 yrs and received Rs 7560 as total interest. Find
the interest amount received by Aakash if the same
amount is invested on compound interest at (r+5)% rate
of interest after 2 yrs?
(a) Rs 7560
(b) Rs 7392
(c) Rs 7120
(d) Rs 7820
(e) Rs 7460

31 www.bankersadda.com | www.sscadda.com | www.careerpower.in | Adda247 App


Daksh Quant Question Bank "June Edition" By Shantanu Shukla
Q329. Satish is 25% less efficient than Abhishek. Bhavya Q334. The circumference of two circles is 132 m and 176
is 100% more efficient than Satish. If Satish, Bhavya m respectively. What is difference between the area of
20
together can complete the work in 3 days, find in how larger circle and area of smaller circle ? (in m²)
many days Bhavya & Abhishek together can complete the (a) 1052
same work. (b) 1128
(a) 5 days (c) 1258
20 (d) 1078
(b) 3 days
(e) 1528
(c) 4 days
(d) 6 days Q335. The perimeter of 4 squares is 24 cm, 32 cm, 40 cm,
17
(e) 3 𝑑𝑎𝑦𝑠 48 cm respectively. What will be the area of the square
having perimeter equal to sum of edges of 4 squares? (in
Q330.P can do a piece of work alone in 30 days. If P and Q sq.cm.)
2
together can do rd of same work in 8 days, the find in how (a) 64
3
3 (b) 81
many days Q alone can complete 4th of the same work?
(c) 100
(a) 24 days (d) 121
(b) 18 days (e) 144
(c) 12 days
(d) 15 days Q336. If ratio of time periods of investment of P and Q is
(e) None of these 4:5, profit at the end of the year is 75000 and P’s share is
Rs 15000, then what is the ratio of Q’s and P’s
Q331. The parallel sides of a trapezium are 4 cm & 10 cm
investment?
respectively while non-parallel sides are equal to side of
(a)5:16
square of area 25 sq.cm. find area of trapezium. (in
(b) 6:7
sq.cm.)
(c) 12:13
(a) 50
(d) 16:5
(b) 42
(e)8:5
(c) 56
(d) 28
(e) 14 Q337. A and B entered into a partnership with amount Rs
2500 and Rs 4500 respectively and C joined them after 5
Q332. The ratio of area of square to that of rectangle of months with amount Rs 2400 and if total profit at the end
length 10 cm is 4 : 5. If breadth of rectangle is same as of the year is Rs 16800, then find the difference between
side of square. Find length of diagonal of square. profit amount earned by B and C ?(in Rs)
(a) 9√2 𝑐𝑚 (a) 6000
(b) 5800
(b) 10√2 𝑐𝑚
(c) 5600
(c) 6√2 𝑐𝑚
(d) 6200
(d) 4√2 𝑐𝑚 (e) 6400
(e) 8√2 𝑐𝑚
Q338. Arun, Bhavya & Ashu entered into a partnership,
Q333. The total surface area of a cylinder is 368π 𝑐𝑚2 ratio of investment of Arun & Bhavya is 4 : x and ratio of
and sum of radius and height of cylinder is 23cm. Find the investment of Bhavya & Ashu is 3 : 4. If at the end of two
volume of cone whose total surface area is 200𝜋 𝑐𝑚2. years Ashu receives Rs 1850 as profit out of total profit Rs
(radius of cylinder and cone is equal) 3700. Then find the value of x?
(a) 512π 𝑐𝑚3
(a) 12
(b) 640π 𝑐𝑚3 (b) 14
(c) 320π 𝑐𝑚3
(c) 16
(d) 290π 𝑐𝑚3
(d) 8
(e) 400π 𝑐𝑚3
(e) 10

32 www.bankersadda.com | www.sscadda.com | www.careerpower.in | Adda247 App


Daksh Quant Question Bank "June Edition" By Shantanu Shukla
Q339. A, B and C invested Rs 7000, Rs 6000 and Rs 8500 Q344.The speed of the boat in still water in 15 km/hr. If
respectively in a partnership for 2 yrs .After 2 yrs, A and B the boat travels 54 km each in downstream and upstream
increased their investment by Rs 2000 and Rs 1500 in 7.5 hrs, then find the time taken by the boat to travel 48
respectively and C decreased his investment by Rs 2000. km in upstream?
At the end of three yrs, total profit received by them is Rs (a) 8 hrs
26400, then find B’s profit share? (b) 6 hrs
(a) Rs 8400
(c)3 hrs
(b) Rs 9200
(d) 5 hrs
(c) Rs 7200
(d) Rs 7800 (e) 4 hrs
(e) Rs 8000
Q345.A boat which takes 6 hr to travel 105 km in still
Q340. A and B started a business with some amount. water, goes 364 km in upstream and return back to the
9
After 9 months B left the business & C joins the business initial point. If rate of stream is th of upstream speed of
26
with Rs. 12,000 and remains in business till the end of
boat, then find how much approximate time did it take in
year. At the end of the year, profit share of A, B and C is Rs.
the entire journey?
48, Rs. 48 and Rs. 24 respectively. Find the sum of the
amount (in Rs.) invested by A and B together in the (a) 48 hrs
business? (b) 40 hrs
(a) 8,000 (c) 52 hrs
(b) 10,000 (d) 45 hrs
(c) 15,000 (e) 56 hrs
(d) 12,000
(e) 14,000 Q346. The ratio of income of A, B and C is 3 : 7 : 4 and
ratio of expenditure is 4 : 2 : 5. If A saves Rs 300 out of Rs
Q341. A boat covers a distance of 10.8 km upstream in 2700, find the average expenditure of A, B and C (in Rs).
36minutes and the speed of boat in still water is 21kmph. (a) 2200
Find the time taken by boat to cover 60km downstream?
(b) 2090
(a) 2 hours 15 minutes
(c) 2321
(b) 2 hours 30 minutes
(c) 1 hour 48 minutes (d) 2000
(d) 2 hours (e) 1800
(e) 2 hours 40 minutes
Q347. There are 250 questions in an exam. Ram gets 3
Q342.A boat covers 36 km in downstream in 4 hrs. if the marks for every correct answer and
1
speed of the current is 3rd of its downstream speed, then –0.5 marks for every wrong answer. If he attempted all
in what time will it cover a distance of 78 km upstream? the questions and scored 435 marks, find the no. of
(a) 30 hrs questions he attempted correct?
(b) 26 hrs (a) 162
(c) 28 hrs (b) 164
(d) 24 hrs (c) 168
(e) 32 hrs (d) 160
(e) 150
Q343. A boat goes 220 km downstream and 108 km
upstream in 20 hr. Speed of the boat in still water is 4 Q348. In a committee of 20 members, the average age is
times the speed of the stream. Find the sum of time taken 25 years. The average age of first 18 members is 24 years.
by the boat to go 40 km in downstream and 48 km
What will be the average age of last 2 members?
upstream?
(a) 8 hrs (a) 32
(b) 10 hrs (b) 36
(c) 6 hrs (c) 38
(d) 9 hrs (d) 34
(e) None of these (e) 30

33 www.bankersadda.com | www.sscadda.com | www.careerpower.in | Adda247 App


Daksh Quant Question Bank "June Edition" By Shantanu Shukla
Q349. Average expenditure of Manoj & Nawaz is Rs 4500 Q353. what is average of students who passed in only one
which is 10% less than that of Sanjay & Irfan. If Sanjay examination in all sections together?
spends Rs 500 more than Nawaz & average expenditure (a) 39.67
of Nawaz & Sanjay is Rs 4250. Find average expenditure (b) 40.67
of Manoj & Irfan. (in Rs) (c) 41.67
(a) 4250 (d) 42.67
(b) 5000 (e) 43.67
(c) 4750
(d) 5250 Q354. Students failed in both exams in section C are what
(e) 4500 percent of total students in section C? (in %)
(a) 30
Q350. Anurag ordered three burgers for Rs. 200, Deepak
(b) 20
ordered 2 burgers of average price Rs 80 & Veer ordered
(c) 18
3 burgers, each burger cost him Rs 95. Find average cost
(d) 25
of each burger.
(a) 𝑅𝑠. 85.625 (e) 33.33
(b) 𝑅𝑠. 75.625
(c) 𝑅𝑠. 70.625 Q355. Which sections have equal number of students?
(d) 𝑅𝑠. 105.65 (a) section A & B
(e) 𝑅𝑠 80.625 (b) section A & C
(c) section B & C
Directions (151-155):- Given table shows the data of (d) all have same no.of students
students of a class related to results of Half-yearly and (e) none
Annual examination. Study the data carefully and answer
the questions. Directions (356-360): Following Table chart gives the
details of 5 students of a particular school in five different
Section Section Section subjects in the annual exam.
A B C Maths Physics Chemistry English Computer
Students who have (150) (150) (150) (100) (100)
10 15 20 Amit 70 66 58 54 80
failed in both
Aakash 50 64 78 65 75
Students who have
30 30 35 Siddharth 48 72 88 70 86
passed Half-yearly
Lokesh 80 76 84 75 85
Students who have
40 25 30 Ritesh 76 82 64 72 94
passed Annual
Note:-the data provided in the table is percentage of
Students who have
20 20 25 marks out of total marks in that particular subject.
passed in both
Q356. Total marks scored by lokesh in physics, chemistry
Q351. How many students are there in Section B of class?
and maths together is how much more/less than total
(a) 50
(b) 60 marks scored by Amit in the same three subjects
(c) 90 together?
(d) 100 (a) 75
(e) 110 (b) 65
(c) 69
Q352. Students passed in both exams in all sections are (d) 55
what percent more/less than students failed in both (e) 80
exams in all sections?
10
(a) 44 % Q357. Find the overall percentage of marks scored by
13
10 Siddharth in the exam?
(b) 30 % (a) 75%
13
(c) 40% (b) 82%
4
(d) 44 9 % (c) 68%
4
(e) 40 9 % (d) 72%
(e) 80%

34 www.bankersadda.com | www.sscadda.com | www.careerpower.in | Adda247 App


Daksh Quant Question Bank "June Edition" By Shantanu Shukla
Q358. Find the difference of total marks scored by Ritesh Q363. Find average numbers of pens sold by all the three
in all the given subject together and total marks scored by stationary?
Aakash in all the given subjects together? (a) 176.67
(a) 71 (b) 172.67
(b) 84 (c) 177.67
(c) 78 (d) 173.67
(d) 82 (e) 179.67
(e) 93
Q364.If number of pens sold by stationary B is increased
by 20% and number of pencils sold by stationary C is
Q359. Find the average marks scored in physics subject
increased by 25%, then what is sum of total pens sold by
by all the given five students together? stationary B and pencil sold by stationary C?
(a) 105 (a) 312
(b) 110 (b) 322
(c) 108 (c)328
(d) 100 (d) 340
(e) 98 (e) 304

Q360. Total marks scored by Aakash, Siddharth and Q365. What is the difference between total number of
Lokesh in English is what percentage of the total marks pens sold by all the 3 stationary together and total
scored by Amit, Aakash and lokesh in maths? number of pencils sold by all the 3 stationary together?
(a) 75% (a) 178
(b) 70% (b) 172
(c) 65% (c) 168
(d) 68% (d) 184
(e) 80% (e) 190

Direction(366-370): Read the following data carefully


Directions (361-365):- Study the following information
and answer the following question.
carefully and answer the question accordingly.
There are 210 persons in a party, and all of them eat
Three stationary owners A,B and C sells Pen and Pencil. different flavoured icecreams. 40 people eat only
The ratio of the number of pen to pencil sold by stationary butterscotch, 30 people eat all three flavoured
A was 7:5 and that sold by stationary B was 3:2 icecream,there are total 130 people who eat butterscotch
respectively. The number of pens and pencil sold by and 100 people who eat vanila. 40 people eat butterscotch
stationary C was 128 and ratio of number of pen to pencil and vanila only, 10 people eat chocolate and vanila only.
sold by stationary C was 5:3. The total number of pens
sold by stationary A was 10 % more than the pen sold by Q366. What is number of person who eat only Chocolate?
stationary B.Total numbers of pen and pencils sold by all (a) 50
the three stationary was 874. (b) 40
(c) 30
Q361. If cost of each pen and each pencil sold by A is Rs (d) 60
20 and Rs 10 respectively, then find total amount earned (e) 70
by stationary A?
(a) Rs 6370
(b) Rs 6470
(c)Rs 6270
(d) Rs 6300
(e) Rs 6400

Q362.What is the ratio of pens sold by stationary A and B


together to pencils sold by B and C together?
(a) 188:441
(b) 441:188
(c) 233:447
(d) 447:233
(e) None of these

35 www.bankersadda.com | www.sscadda.com | www.careerpower.in | Adda247 App


Daksh Quant Question Bank "June Edition" By Shantanu Shukla
Q367. People eating chocolate and butterscotch only are Q371. What is difference between average marks scored
what percent of people eating only butterscotch? by Student A and Student B in all subjects?
(a) 50%
(a) 1.75
(b) 60%
(c) 25% (b) 1.45
(d) 30% (c) 1.50
(e) 40% (d) 1.25
(e) 1
Q368.number of people eating only vanilla is how much
less than the people eating all three types of icecream ?
(a) 15 Q372. What is Ratio of marks obtained by Student A in
(b) 20 Maths and Computer together to the marks obtained by
(c) 30 Student B in Science and English together?
(d) 10
(a) 7:5
(e) 25
(b) 7:8
Q369. People eating chocolate are what percent of people (c) 8:7
eating vanilla icecream? (d) 8:5
(a) 100%
(e) 5:7
(b) 130%
(c) 110%
(d) 120% Q373. What is the overall percentage marks scored by
(e) 90% Student B?
(a) 68.75 %
Q370.what is the ratio of people eating only chocolate
and only butterscotch together to the person eating only (b) 67.5 %
vanilla? (c) 68%
(a) 2:9 (d) 67%
(b) 9:2 (e) 69.25%
(c) 3:7
(d) 7:3
(e) 5:3 Q374. Marks Scored by Student A in Math is what percent
of marks scored by Student B in Science and English
Direction (371-375): Following Line Graph shows the together?
marks scored by Student A and Student B in high school in
(a) 40%
different Subjects. (Maximum Marks is 100 for each
subject). Study the data carefully and answer the (b) 60%
following questions. (c) 50%
100 (d) 70%
90 (e) 80%
80
70
Q375. If passing marks for each subject is 40% of 120,
60
50 then what is the difference between passing marks and
40 marks scored by Student B in Computer?
30
(a) 30
20
10 (b) 32
0 (c) 36
Maths Computer Science English (d) 40
Student A Student B (e) 45

36 www.bankersadda.com | www.sscadda.com | www.careerpower.in | Adda247 App


Daksh Quant Question Bank "June Edition" By Shantanu Shukla
Directions (376-380):- Given line graph shows the sum Q380. Who among the four had received the highest
invested, rate of interest and time period of investment by amount as interest?
4 people. Study the data carefully and answer the (a) Karan
questions. (b) Anurag
(NOTE: all invested their sum at simple interest) (c) Both Anurag & Mahesh
(d) Rohit
16 (e) Mahesh
14
Directions (381-385):- Given bar graph shows the data
12 of expenses (in % distribution) of Mr. Chunky in 4 months
on rent, travel & food. Study the graph carefully and
10 answer the questions.
8 100%
6 90%
4
80%
2
70%
0
Karan Anurag Mahesh Rohit 60%
Food
50%
Sum (Rs.) (in 1000s) Rate (%) Time (years) Travel
40%
Rent
Q376. How much will Rohit receive after completion of 30%
his investment period? (in Rs.) 20%
(a) 5200
(b) 6800 10%
(c) 4800
0%
(d) 4400
March May June July
(e) 4600

Q377. Interest amount received by Mahesh is what Q381. If salary of Mr. Chunky is Rs. 12000 in July and his
percent more than interest amount received by Karan? savings is half of his expenditure on rent. Find his
(a) 85% expenditure on food. (in Rs. )
(b) 60% (a) 3500
(c) 75% (b) 2000
(c) 4000
(d) 70%
(d) 3000
(e)80%
(e) 4500
Q378. What is total amount received as interest by Q382. If savings and salary of Mr. Chunky are same for all
Anurag & Rohit together? (in Rs.) the given months, then expenditure on travel in March is
(a)None of these what percent of expenditure on food in June?
(b) 3150 (a) 87.5%
(c) 3200 (b) 85%
(d) 3360 (c) 90%
(e) 3420 (d) 92.5%
(e) 𝑁𝑜𝑛𝑒 𝑜𝑓 𝑡ℎ𝑒𝑠𝑒
Q379. If Karan had invested same sum at compound
interest at same rate of interest for same period. How Q383. If ratio of total expenditure in May & July is 5:4.
much more would he earn? Find ratio of expenditure on rent in May to expenditure
on travel in July.
(a)Rs 80 (a) 3:2
(b)Rs 90 (b) 6:7
(c) Rs 70 (c) 7:6
(d) 𝑅𝑠 60 (d) 24: 35
(e) None of these (e) 35: 24

37 www.bankersadda.com | www.sscadda.com | www.careerpower.in | Adda247 App


Daksh Quant Question Bank "June Edition" By Shantanu Shukla
Q384. Income of Chunky in March & July is Rs. 5000 & Rs. Q387. Nokia mobiles produced in 2016 & 2017 together
8000 of which he saves only 10% in each month. What is are how much more than Samsung mobiles produced in
his average expenditure on rent in these 2 months? 2018 & 2019?
(a)800
(a) 𝑅𝑠. 2400 (b) 100
(b) 𝑅𝑠. 2300 (c) 400
(c) Rs. 2340 (d) 300
(d) 𝑅𝑠. 2430 (e)200
(e) 𝑅𝑠. 2360
Q388. Samsung mobiles produced in 2018 are what
percent of Nokia mobiles produced in 2019?
Q385. Expenditure on travel in May is what percent more (a)None of these
than expenditure on travel in July if total expenditure for (b) 60%
both the months is same? (c) 75%
2
(a) 15% (d) 66 3 %
2
(b) 12.5% (e) 68 %
3
(c) 16.67%
(d) 20% Q389. What is the ratio of Nokia mobiles produced in
(e) 10% 2016, 2017 & 2018 together to Samsung mobiles
produced in 2016, 2017 & 2019 together?
(a) 83:96
Directions (386-390): Given bar graph shows the
(b) 35:32
production of mobile phones by Nokia & Samsung in 4 (c) 83:86
years. Study the data carefully and answer the questions. (d) 96: 83
(e) None of these
4.5
No. of mobile phones produced (in 1000s)

Q390. In which year the increase in production was


4 maximum as compared to previous year & for which
company?
(a) Nokia, 2017
3.5 (b) Nokia, 2018
(c) Samsung, 2019
3 (d) Nokia, 2019
(e) Samsung, 2017
2.5
Directions (391-395): Given pie diagram shows the
percentage distribution of number of registered voters in
2 5 villages. Study the diagram carefully and answer the
following questions.
1.5 No. of registered voters = 10000

1
2016 2017 2018 2019 E A
20% 20%
Nokia Samsung

Q386. How many mobile phones have been produced of


D
Samsung over all the years?
15%
(a)10800 B
C 25%
(b) 11600 20%
(c) 11400
(d) 11000
(e)11200

38 www.bankersadda.com | www.sscadda.com | www.careerpower.in | Adda247 App


Daksh Quant Question Bank "June Edition" By Shantanu Shukla
Q391. If 20% of registered voters in village B did not cast Directions (396-400): Study the charts given below
their vote and 10 % of votes cast were found invalid. Find carefully and answer the following questions.
no. of valid votes cast in village B. Pie chart shows the percentage distribution of total
(a) 1800 employee in 5 different companies as shown below and
table shown below shows the ratio of males to females in
(b) 1900
these 5 companies.
(c) 1950
(d) 1850 Total employee = 5400
(e) 2000

Q392. In village C, 10% of registered voters did not cast E A


22% 18%
their vote and no vote was invalid from the votes which
were cast. The winning candidate defeated the other
candidate by 12% of votes cast. Find no. of votes obtained
by losing candidate. (There are only 2 candidates
contesting in elections in village C) D B
(a) 996 20% 28%
(b) 880
(c) 1008 C
12%
(d) 792
(e) None of these
Ratio of total males to females (M: F)
Q393. Find average number of registered voters in village A 2:1
B, C & D. B 3:1
C 1:2
(a) 1700
D 2:3
(b) 2100
E 2:1
(c) 1900
(d) 1800 Q396.What is the ratio of number of males in company E
(e) 2000 to the number of females in company D?
(a) 7: 11
Q394. In village A, B, D & E votes cast by only 70%, 65%, (b) 9:11
80% and 75% of registered voters respectively. From (c) 11:9
which village among A, B, D & E did maximum voters cast (d) 11: 7
(e) 7:13
their votes?
(a) 𝐸
Q397.Total number of males in company A are
(b) 𝐴 approximately what percent of total females in company
(c) B E?
(d) 𝐷 (a) 164%
(e) 𝐵 & 𝐸 (b) 152%
(c) 170%
Q395. Average no. of registered voters from village A & C (d) 144%
(e) 138%
is what percent of average no. of registered voters from
village B, D and E?
Q398.Total males in B, C & D together are what percent of
(a) 120% total employees in all 5 companies together?
(b) 100% (a) 38%
(c) 90% (b) 33%
(d) 80% (c) 45%
(e) 110% (d) 48%
(e) 52%

39 www.bankersadda.com | www.sscadda.com | www.careerpower.in | Adda247 App


Daksh Quant Question Bank "June Edition" By Shantanu Shukla
Q399.How many females employee are there in all the 5
companies together?
(a) 2084
(b) 2304
(c) 2256
(d) 2178
(e) 2280

Q200. Find the central angle of total employees from


company’s B and D together?
(a) 151.2°
(b) 162°
(c) 165.6°
(d) 187.2°
(e) 172.8°

Solutions
S1. Ans.(d) S6. Ans.(a)
Sol. Sol.

S7. Ans.(b)
S2. Ans.(b)
Sol.
Sol.

S8. Ans.(c)
S3. Ans.(e)
Sol.
Sol.

S9. Ans.(a)
Sol.

S4. Ans.(a)
Sol.

S10. Ans.(e)
Sol.

S5. Ans.(e)
Sol.
S11. Ans.(a)
Sol.

40 www.bankersadda.com | www.sscadda.com | www.careerpower.in | Adda247 App


Daksh Quant Question Bank "June Edition" By Shantanu Shukla
S12. Ans.(c)
Sol.

S13. Ans.(d)
Sol.
S19. Ans.(a)
Sol.

S14. Ans.(a)
Sol. S20. Ans.(c)
Sol.

S15. Ans.(e)
Sol.

S16. Ans.(d)
Sol.

S21. Ans(d)
Sol.

S17. Ans.(a)
Sol.

S22. Ans(a)
Sol.

S18. Ans.(d)
Sol.
S23. Ans.(d)
Sol.

41 www.bankersadda.com | www.sscadda.com | www.careerpower.in | Adda247 App


Daksh Quant Question Bank "June Edition" By Shantanu Shukla
S24. Ans(a) S30. Ans(b)
Sol. Sol.

S25. Ans(d)
Sol.

S31. Ans(b)
Sol.

S26. Ans(b)
Sol.
S32. Ans(d)
Sol.

S27. Ans(d)
Sol. S33. Ans(a)
Sol.

S34. Ans(e)
Sol.

S28. Ans(a)
Sol. S35. Ans(c)
Sol.

S36.Ans.(d)
Sol. Required difference = average marks scored by
S29. Ans(c) Student A - Average marks scored by Student B
Sol. Total number of students who have opted for engg.
stream in college R

S37.Ans(c)
Sol.

42 www.bankersadda.com | www.sscadda.com | www.careerpower.in | Adda247 App


Daksh Quant Question Bank "June Edition" By Shantanu Shukla
S38. Ans(b) S45. Ans.(d)
Sol. Sol.

S39. Ans.(c)
Sol.

S46. Ans(e)
S40. Ans.(b) Sol.
Sol.

S41. Ans. (c)


Sol.

S47. Ans.(c)
Sol.

S42. Ans.(a)
Sol.

S48. Ans.(b)
Sol.

S43. Ans.(d)
Sol.

S49. Ans(a)
Sol.

S44. Ans.(e)
Sol.

S50. Ans(d)
Sol.

43 www.bankersadda.com | www.sscadda.com | www.careerpower.in | Adda247 App


Daksh Quant Question Bank "June Edition" By Shantanu Shukla
S51. Ans(d) S57. Ans.(d)
Sol. Sol.

S52. Ans(a) S58. Ans.(a)


Sol. Sol.

S59. Ans.(e)
Sol.
S53. Ans(e)
Sol.

S60. Ans.(b)
Sol.
S54. Ans(d)
Sol.

S61. Ans.(a)
Sol.
S55. Ans(a)
Sol.

S62. Ans.(c)
Sol.

S56. Ans(b)
Sol.
S63. Ans.(b)
Sol.

44 www.bankersadda.com | www.sscadda.com | www.careerpower.in | Adda247 App


Daksh Quant Question Bank "June Edition" By Shantanu Shukla
S64. Ans.(e) S70. Ans.(e)
Sol. Sol.

S71. Ans.(b)
S65. Ans.(d) Sol.
Sol.

S66. Ans.(b)
Sol.

S67. Ans.(d) S72. Ans.(a)


Sol.
Sol.

S68. Ans.(c)
Sol.

S69. Ans.(e)
Sol.

S73. Ans. (b)


Sol.

S74. Ans.(c)
Sol.

45 www.bankersadda.com | www.sscadda.com | www.careerpower.in | Adda247 App


Daksh Quant Question Bank "June Edition" By Shantanu Shukla
S80. Ans.(c)
Sol.

S75. Ans.(d)
Sol.

S81.Ans.(c)
Sol.

S76. Ans.(e)
Sol.

S82. Ans.(a)
Sol.

S83. Ans(b)
Sol.
S77. Ans.(a)
Sol.

S84. Ans(a)
S78. Ans.(c) Sol.
Sol.

S85. Ans(b)
S79. Ans.(a) Sol.
Sol.

46 www.bankersadda.com | www.sscadda.com | www.careerpower.in | Adda247 App


Daksh Quant Question Bank "June Edition" By Shantanu Shukla
S86. Ans(d)
Sol.

S91. Ans.(d)
Sol.

S87. Ans(c)
Sol.

S92. Ans.(a)
Sol.

S88.Ans(c)
Sol.

S93. Ans.(c)
Sol.

S94. Ans.(e)
Sol.

S89. Ans(a)
Sol.

S95. Ans.(d)
Sol.

S90. Ans(d)
Sol.

47 www.bankersadda.com | www.sscadda.com | www.careerpower.in | Adda247 App


Daksh Quant Question Bank "June Edition" By Shantanu Shukla
S96. Ans.(b)
Sol.

S103. Ans.(d)
Sol.

S97. Ans.(c)
Sol.

S98. Ans.(a)
Sol.

S104. Ans.(e)
Sol.

S99. Ans.(e)
Sol.

S105. Ans.(a)
S100. Ans.(d) Sol.
Sol.

S101. Ans.(d)
Sol.

S102. Ans.(d)
Sol.
S106.Ans(a)
Sol. Total students in a section = students failed in both +
students passed in half yearly + students passes in annual
– students passed in both
total students in section B = 15+30+25-20=50

48 www.bankersadda.com | www.sscadda.com | www.careerpower.in | Adda247 App


Daksh Quant Question Bank "June Edition" By Shantanu Shukla
S107. Ans(d) S113. Ans(e)
Sol. Sol.

S108. Ans(c)
Sol.
S114. Ans(c)
Sol.

S109. Ans(e)
Sol.

S115. Ans(b)
S110. Ans(b) Sol.
Sol.

S111. Ans(b)
Sol. S116. Ans(c)
Sol.

S117. Ans.(d)
Sol.

S112. Ans(d)
Sol.

S118. Ans.(b)
Sol.

49 www.bankersadda.com | www.sscadda.com | www.careerpower.in | Adda247 App


Daksh Quant Question Bank "June Edition" By Shantanu Shukla
S119. Ans(e) S126. Ans.(b)
Sol. Sol.

S127. Ans.(d)
Sol.

S120. Ans(b)
Sol.
S128. Ans.(e)
Sol.

S129. Ans.(a)
S121. Ans.(e)
Sol.
Sol.

S122. Ans.(a) S130. Ans.(c)


Sol. Sol.

S123. Ans.(b)
Sol.

S124. Ans.(d) S131. Ans.(c)


Sol. Sol.

S125. Ans.(e) S132. Ans.(e)


Sol. Sol.

50 www.bankersadda.com | www.sscadda.com | www.careerpower.in | Adda247 App


Daksh Quant Question Bank "June Edition" By Shantanu Shukla
S133. Ans.(a) S138. Ans.(d)
Sol. Sol.

S139. Ans.(e)
Sol.

S134. Ans.(d)
Sol.

S140. Ans.(c)
Sol.

S135. Ans.(c)
Sol.

S141. Ans(b)
Sol.

S136. Ans.(b)
Sol.

S137. Ans.(b)
Sol.
S142. Ans.(d)
Sol.

51 www.bankersadda.com | www.sscadda.com | www.careerpower.in | Adda247 App


Daksh Quant Question Bank "June Edition" By Shantanu Shukla
S143. Ans.(a) S149. Ans.(c)
Sol. Sol.

S144. Ans.(d) S150. Ans.(b)


Sol. Sol.

S151. Ans.(b)
Sol.

S145. Ans.(a)
Sol.
S152. Ans.(d)
Sol.

S153. Ans.(d)
S146. Ans.(b) Sol.
Sol.

S147. Ans.(b)
Sol.

S154. Ans.(c)
Sol.
S148. Ans.(d)
Sol.

52 www.bankersadda.com | www.sscadda.com | www.careerpower.in | Adda247 App


Daksh Quant Question Bank "June Edition" By Shantanu Shukla
S162. Ans.(e)
Sol.

S155. Ans.(b)
Sol.

S163. Ans.(d)
Sol.

S156. Ans.(d)
Sol. S164. Ans.(a)
Sol.

S165. Ans.(e)
S157. Ans.(d) Sol.
Sol.

S158. Ans.(a)
Sol.

S166. Ans.(c)
Sol.

S159. Ans.(d)
Sol.

S167. Ans.(d)
Sol.

S160. Ans.(e)
Sol.

S168. Ans.(a)
Sol.
S161. Ans.(c)
Sol.

53 www.bankersadda.com | www.sscadda.com | www.careerpower.in | Adda247 App


Daksh Quant Question Bank "June Edition" By Shantanu Shukla
S175. Ans.(b)
Sol.

S169. Ans.(c)
Sol.

S170. Ans.(b)
Sol. S176. Ans.(c)
Sol.

S171. Ans.(d) S177. Ans.(d)


Sol. Sol.

S172. Ans.(c) S178. Ans.(a)


Sol. Sol.

S173. Ans.(b)
Sol.

S179. Ans.(b)
Sol.

S174. Ans.(e)
Sol.

54 www.bankersadda.com | www.sscadda.com | www.careerpower.in | Adda247 App


Daksh Quant Question Bank "June Edition" By Shantanu Shukla
S180. Ans.(e)
Sol.

S186. Ans.(d)
Sol.

S181. Ans.(b)
Sol.

S187. Ans.(e)
Sol.

S182. Ans.(c)
Sol.

S183. Ans.(e) S188. Ans.(a)


Sol. Sol.

S184. Ans.(d)
Sol.

S185. Ans.(b)
Sol.

55 www.bankersadda.com | www.sscadda.com | www.careerpower.in | Adda247 App


Daksh Quant Question Bank "June Edition" By Shantanu Shukla
S189. Ans.(d) S193. Ans (e)
Sol. Sol.

S194. Ans (d)


Sol.

S190. Ans.(a)
Sol.

S195. Ans (a)


Sol.

S191. Ans (c)


Sol.

S196. Ans.(d)
Sol.

S192. Ans (e)


Sol.

56 www.bankersadda.com | www.sscadda.com | www.careerpower.in | Adda247 App


Daksh Quant Question Bank "June Edition" By Shantanu Shukla
S197. Ans(c)
Sol.

S201. Ans.(d)
48 ?
Sol. 100 × 525 + 100 × 250 = 499
247×100
?= = 98.8
250

S202. Ans.(c)
5 7 1
Sol. × × × 1600 = 260 + ? – 499
2 8 28
? = 499 + 125 – 260 = 364

S198. Ans(a) S203. Ans.(a)


Sol. Sol. ? = √5125 – 289 – 75
= √4761 = 69

S204. Ans.(b)
Sol. (?)² = 16 × 7 + 361 + 11
= 484
? = 22.

S205. Ans.(b)
Sol. 252 + 26 + 420 = 121 + ?
? = 577

S206. Ans.(c)
40×8500
Sol. 80% 𝑜𝑓 ? = √250 × 44 +
100
80
⇒ 100
× ? = √11000 + 3400
10
S199. Ans(e) ⇒? = √14400 × 8
Sol. ⇒ ? = 120 ×
10
= 150
8

S207. Ans.(a)
40 594 2300
Sol. ?× 24 × 27 = 115 × 264
⇒ ? × 45 = 45 ⇒ ? = 1

S200. Ans(e)
Sol.

57 www.bankersadda.com | www.sscadda.com | www.careerpower.in | Adda247 App


Daksh Quant Question Bank "June Edition" By Shantanu Shukla
S208. Ans.(d) S214. Ans.(a)
20 4 14 3 4
Sol. 100 × 40 × √? = 322 + 16² Sol. (7 × 5 ÷ 2) − (10 𝑜𝑓 ? ) = 5 − 3
1 4 14 1 3 11
⇒ √? = × (1024 + 256) (7 × × 2) − (10 ×? ) = −
8 5 5
1 4 3 11
⇒ √? = × 1280 = 160 − 10 ? = −
8 5 5
⇒ ? = (160)² = 25600 ? = 10

S209. Ans.(b) S215. Ans.(c)


45
Sol. ? +13 × 50 = 420 + 100 × 800 + 220 4 1 27 1
Sol. 4 + 2 − = 2 ÷ 3 ×?
5 15 5 5
⇒ ? +650 = 420 + 360 + 220 24 31 27 11 1
+ 15 − = × 3 ×?
⇒ ? = 1000 − 650 = 350 5 5 5
22 11
= ×?
15 15
S210. Ans.(e) ?= 2
3
Sol. (? )2 = 256 × (2)8 ÷ (8)5 × 32
S216. Ans.(e)
3
28 ×28
⇒ (? )2 = 215
× 25
3 Sol. √5776 - √1444 + √729 =43 + ?
⇒ (? ) = 2 (2)6 = 64 76 – 38 +27 =43 + ?
2
⇒ ? = (64) = 16 3 ?=65 -43 =22

S211. Ans.(c) S217. Ans.(a)


4
4 𝑜𝑓25 5 3 1 Sol. 78 ×26÷6 +1262= 1311 + (?)2
Sol. ( 5
48
) ÷ (4 𝑜𝑓32 + 7 𝑜𝑓 21) =? 𝑜𝑓 49 2028÷6+1262 =1311 +(?)2
24 25 1 338+1262 =1311+(?)2
( 5 × 48) ÷ (40 + 9) =?× 49 (?)2=1600 -1311 =289
5 5
? = 49 × 98 = 2 = 2.5 ? =√289 =17

S212. Ans.(b) S218. Ans.(a)


1
Sol. √? 𝑜𝑓 6 + 20% 𝑜𝑓 95 = 2 of 62 Sol. 1484÷28 + 1462÷34 -12×7= ?
62 20 ?=53+43 -84 = 12
√? 𝑜𝑓 6 = 2

100
× 95 = 12
? = 22 = 4 S219. Ans.(c)
Sol. 42.5×15 +37.5× 25= 1420 + ?
S213. Ans.(e) 637.5+937.5 =1420 + ?
5 3
Sol. ( 𝑜𝑓 6 𝑜𝑓
9
) +?2 = 45 ?= 1575 – 1420 = 155
3 5 11
5 33 9 2
(3 × 5 × 11) +? = 45 S220. Ans.(b)
?2 = 36 Sol. 2450 +3760 -3830 =6000 - ?
? = ±6 2380 =6000 - ?
?=6000 -2380 = 3620

S221. Ans.(a)
125.98 198.02 156.05 51.03
Sol. 154.03 × 17.99 − 101.98 × 78.03 = ?
126 198 156 51
154
× 18
− 102 × 78 ≈?
?≈ 9−1 ≈ 8

S222. Ans.(d)
Sol. 80.08% of 349.98 + 45.02% of 799.99 =
? % × 255.95
80% 𝑜𝑓 350 + 45% 𝑜𝑓 800 ≈? % × 256
280 + 360 ≈? % × 256
640
? ≈ 256 × 100 = 250

58 www.bankersadda.com | www.sscadda.com | www.careerpower.in | Adda247 App


Daksh Quant Question Bank "June Edition" By Shantanu Shukla
S223. Ans.(b) S233. Ans.(a)
Sol. √1224.99 ÷ 6.99 = ? – 1799.98 Sol. √1443.979 ÷ 18.981 + 3.5 × √16.017 = (? )
√1225 ÷ 7 ≈? −1800 √1444 ÷ 19 + 3.5 × √16 ≈?
5 ≈? −1800 38
? ≈ + 3.5 × 4
? ≈ 1810 19
? ≈ 2 + 14 ≈ 16
S224. Ans.(e)
Sol. 2744.98 – 1417.99 = ? + 987.98 S234. Ans.(e)
2745 − 1418 ≈? +988 Sol. 779.98 ÷ 48.014 × 15.989 = ?
780
? ≈ 339 48
× 16 ≈?
780
?≈ ≈ 260
S225. Ans.(c) 3

Sol. ?2 = 44.99 % of 4500.02– 24.99% of 3959.98 +


S235. Ans.(b)
87.01 × 2.97
?2 ≈ 45% 𝑜𝑓 4500 − 25% 𝑜𝑓 3960 + 87 × 3 Sol. 1485.988 + 212.04 − 1703.99 = ? −(11.02)2
?2 ≈ 1296 1486 + 212 − 1704 ≈ ? – (11)2
? ≈ 36 ? ≈ 1698 − 1704 + 121 ≈ 115

S226. Ans.(a) S236. Ans.(d)


Sol. 1749.98 ÷ 350 × 49.79 + 111.03 = (? )² 64.02 1
Sol. 43.495 × 31.99 × 28.979 – 2.012 =?
1750
× 50 + 111 ≈ (? )2 64 1
350 43.5 × × − 2 ≈?
? = 19 32 29
?≈ 1
S227. Ans.(a)
Sol. ? × 625.04 = 15625.01 + 9999.99 S237. Ans.(b)
?× 625 ≈ 15625 + 10000 Sol. (33.33 × 80.989 ÷ 99.99) + 3.024−? = 4.012
? ≈ 41 (
33.33
× 81) + 3−? ≈ 4
99.99
S228. Ans.(c) ? ≈ 26
Sol. 29.98% of 701 – 350.01 + 82% of 501 = ?
30% 𝑜𝑓 700 − 350 + 82% 𝑜𝑓 500 ≈? S238. Ans.(a)
? ≈ 210 − 350 + 410 ≈ 270 Sol. 20.021 + 4.969 + 30.499 − 50.022 =?
20 + 5 + 30.5 − 50 ≈?
S229. Ans.(e) ? ≈ 5.5
Sol. 5759.99 ÷ 45.01 + 11.99 = ?× 10.03
5760 ÷ 45 + 12 ≈ ? × 10
140 S239. Ans.(c)
? ≈ 10 ≈ 14 Sol. 995.013 − 39.976 × 19.99 + 5.022 = 1.988 ×?
995 − 40 × 20 + 5 = 2 ×?
S230. Ans.(c) ? ≈ 100
Sol. 1395.98 + 412.04 − 2703.99 = ? −(31.02)2
1396 + 412 − 2704 ≈ ? −(31)2
S240. Ans.(e)
? ≈ 961 − 896 ≈ 65
Sol. (10.011)2 + (23.989)2 = 275.99 +?2
S231. Ans.(d) 102 + 242 = 276 +?2
22
Sol. 41.979 × 7 + 19.989% of 530.014 – 26.021 =? ? = 20
22
42 × 7 + 20% 𝑜𝑓 530 − 26 ≈? S241. Ans.(b)
? ≈ 132 + 106 − 26 ≈ 212 Sol. Pattern is
0.5 × (2 − 0) = 1
S232. Ans.(c)
1 × (2 − 0.5) = 1.5
Sol. (23.012 × 22.989) + 20.985 × 7.014 = ?2
(23 × 23) + 21 × 7 ≈?2 1.5 × (2 − 1) = 1.5
?2 ≈ 529 + 147 ≈ 676 1.5 × (2 − 1.5) = 0.75
? ≈ 26 0.75 × (2 − 2) = 0

59 www.bankersadda.com | www.sscadda.com | www.careerpower.in | Adda247 App


Daksh Quant Question Bank "June Edition" By Shantanu Shukla
S242. Ans.(d) S249. Ans.(d)
Sol. Pattern is Sol.
5 × 3 = 15
15 × 3 = 45
45 × 3 = 135
135 × 3 = 405 S250. Ans.(b)
405 × 3 = 1215 Sol.

S243. Ans.(e)
Sol. Pattern is
90 + 6 = 96; 96 + 6 = 102
102 + 6 = 108; 108 + 6 = 114
114 + 6 = 120 S251. Ans.(d)
Sol.
S244. Ans.(a)
Sol. Pattern is
389 − (9 + 0) = 380
380 − (9 + 1) = 370
370 − (9 + 2) = 359
359 − (9 + 3) = 347
347 − (9 + 4) = 334 S252. Ans.(a)
Sol.
S245. Ans.(b)
Sol. Pattern is addition of prime no.
1+2 = 3
3+3 = 6
S253. Ans.(b)
6 + 5 = 11
Sol.
11 + 7 = 18
18 + 11 = 29

S246. Ans.(c)
Sol. S254. Ans.(e)
Sol.

S255. Ans.(c)
S247. Ans.(e) Sol.
Sol.

S256. Ans.(d)
S248. Ans.(a) Sol. addition of prime numbers
Pattern is
Sol.
31 + 2 = 33
33 + 3 = 36
36 + 5 = 𝟒𝟏
41 + 7 = 48
48 + 11 = 59

60 www.bankersadda.com | www.sscadda.com | www.careerpower.in | Adda247 App


Daksh Quant Question Bank "June Edition" By Shantanu Shukla
S257. Ans.(e) S262. Ans.(d)
Sol. Pattern is Sol. Pattern is
6 × 6 = 36 2000× 1 = 2000
36 × 5 = 180 2000÷ 2 = 1000
180 × 4 = 720 1000× 3 = 3000
720 × 3 = 𝟐𝟏𝟔𝟎 3000÷ 4 = 750
2160 × 2 = 4320 750× 5 = 3750
3750 ÷ 6 = 625
S258. Ans.(b) wrong number is 600 which should be replaced with 750
Sol. Pattern is
23 + 6 = 29 S263. Ans.(a)
29 + 6 = 𝟑𝟓
Sol. Pattern is
35 + 6 = 41
2 × 1 + 0=2
41 + 6 = 47
47 + 6 = 53 2 × 2 + 1=5
5 × 3 + 2=17
S259. Ans.(d) 17 × 4 + 3=71
Sol. 1 + 22 = 5 71 × 5 + 4=359
5 + 32 = 𝟏𝟒 359 × 6 + 5=2159
14 + 42 = 30 wrong number is 72 which should be replaced with 71
30 + 52 = 55
55 + 62 = 91 S264. Ans.(e)
Sol. Pattern is
S260. Ans.(c) 9000 –(180× 6)=7920
Sol. Pattern is 7920 –(180× 5)=7020
5 + (5 × 1) = 10 7020 –(180× 4)=6300
10 + (5 × 2) = 20 6300 –(180× 3)=5760
20 + (5 × 3) = 35 5760 –(180× 2)=5400
35 + (5 × 4) = 55 5400 –(180× 1)=5220
55 + (5 × 5) = 𝟖𝟎 wrong number is 5200 which should be replaced with
5220
S261. Ans.(b)
Sol. Pattern is S265. Ans.(d)
102+10=110 Sol. Pattern is
122+12=156 100 + (4× 5) =120
142+14=210 120 + (5× 6) =150
162+16=272
150 + (6× 7) =192
182+18=342
192 + (7× 8) =248
202+20=420
248 + (8× 9) =320
222+22=506
wrong number is 282 which should be replaced with 272 320 + (9× 10) =410
wrong number is 154 which should be replaced with 150

S266. Ans.(c)
Sol. Pattern followed is
7× 0.5 + 0.5 = 4
4× 1 + 1 = 5
5× 1.5 + 1.5 = 9
9× 2 + 2 = 20
20× 2.5 + 2.5 = 52.5
52.5 × 3 + 3 = 160.5
So, wrong number is 8.5 which should be replaced by 9

S267. Ans.(d)
Sol. Pattern followed is
160+47=207
207+53 =260

61 www.bankersadda.com | www.sscadda.com | www.careerpower.in | Adda247 App


Daksh Quant Question Bank "June Edition" By Shantanu Shukla
260+59=319 S273. Ans.(c)
319+61=380 Sol.
380+67=447
447+71=518
So, wrong number is 449 which should be replaced by
447

S268. Ans.(c) So, the wrong no. in this series is 41.


Sol. Pattern followed is
12 × 0.5= 6 S274. Ans.(b)
Sol.
6 × 1= 6
6 × 2 = 12
12 × 3.5= 42
42 × 5.5= 231
So, the wrong no. in this series is 7.
231 × 8= 1848
So, wrong number is 36 which should be replaced by 42 S275. Ans.(d)
Sol.
S269. Ans.(e)
Sol. Pattern followed is
14700 ÷ 7 = 2100
2100× 6 =12600
12600÷ 5 =2520
2520× 4=10080
10080÷ 3 =3360 So, the wrong no. in this series is 53.
3360× 2 =6720
S276. Ans.(d)
So, wrong number is 2500 which should be replaced by
Sol. 10² + 2 = 102
2520
9² + 2 = 83
8² + 2 = 66
S270. Ans.(c) 7² + 2 = 51
Sol. Pattern followed is 6² + 2 = 38
(4.5)2= 20.25 5² + 2 = 27
(4.8)2= 23.04 42 + 2 = 18
(5.1)2= 26.01 Hence, wrong term is 50.
(5.4)2= 29.16
(5.7)2= 32.49 S277. Ans.(c)
(6.0)2= 36.00 Sol. 1² + 1³ = 2
(6.3)2= 39.69 2² + 2³ = 12
So, wrong number is 32.56 and it should be replaced by 3² + 3³ = 36
32.49 4² + 4³ = 80
5² + 5³ = 150
S271. Ans.(d) 6² + 6³ = 252
Sol. 7² + 7³ = 392
So, wrong number is 251

S278. Ans.(c)
Sol. All numbers in the series are prime except 15.
So, the wrong no. in this series is 640
So, wrong term is 15.
S272. Ans.(a) S279. Ans.(a)
Sol. Sol. 11 + 11 = 22
22 + 12 = 34
34 + 13 = 47
47 + 14 = 61
61 + 15 = 76
So, the wrong no. in this series is 1 76 + 16 = 92
So, wrong term is 77

62 www.bankersadda.com | www.sscadda.com | www.careerpower.in | Adda247 App


Daksh Quant Question Bank "June Edition" By Shantanu Shukla
S280. Ans.(a) S285. Ans.(b)
Sol. 2 × 2 + 1 = 5 Sol. I. 𝑥 2 + 41𝑥 + 418 = 0
5 × 2 + 1 = 11 x2+19x+22x+418=0
11 × 2 + 1 = 23 x(x+19)+22(x+19)=0
23 × 2 + 1 = 47 (x+19)(x+22)=0
47 × 2 + 1 = 95 x= -19, -22
95 × 2 + 1 = 191 II. 𝑦 2 + 47𝑦 + 550 = 0
So, wrong term is 6.
y2+22y+25y+550 =0
S281. Ans.(c) y(y+22)+25(y+22)=0
Sol. I. 𝑥 2 − 21𝑥 + 110 = 0 (y+22)(y+25)=0
x2 −11x −10x +110=0 y = -22,-25
x(x-11) −10(x−11)=0 So, x ≥ y
(x-11)(x-10)=0
x =11,10 S286. Ans.(b)
II. 𝑦 2 − 25𝑦 + 156 = 0 Sol. I. 2x² - 17x + 36 = 0
y2-13y -12y +156=0 2x² - 8x – 9x + 36 = 0
y(y-13) -12(y-13)=0 2x(x – 4) – 9 (x – 4) = 0
(y-13)(y-12)=0 (2x – 9) (x – 4) = 0
y= 13,12 9
x= 2 ,4
So, x < y
II. 3y² - 22y + 40 = 0
S282. Ans.(a) 3y² - 12y – 10y + 40 = 0
Sol. I. 𝑥 2 + 29𝑥 + 208 = 0 3y(y – 4) – 10 (y – 4) = 0
x2+16x+13x+208=0 (y – 4) (3y -10) = 0
x (x+16)+13(x+16)=0 y = 4,
10
(x+16) (x+13)=0 3

x= -16, -13 𝑥≥𝑦


II. 𝑦 2 + 35𝑦 + 306 = 0
y2+17y+18y+306 =0 S287. Ans.(c)
y(y+17)+18(y+17)=0 Sol. I. x² + 21x + 108 = 0
(y+18) (y+17)=0 x² + 9x + 12x + 108 = 0
y = -17, -18 x(x + 9) + 12 (x + 9) = 0
So, x > y (x + 12) (x + 9) = 0
x= -12, -9
S283. Ans.(b) II. y² + 14y + 48 = 0
3
Sol. I. x = √4096 y² + 6y+ 8y+ 48 = 0
x= 16 y(y + 6) + 8 (y + 6) = 0
II. (y + 8) (y + 6) = 0
y2 + 121 = 377 y = -8, -6
y2 = 256 y>x
y = ± 16
So, x ≥ y

S284. Ans.(e)
Sol. I. 3𝑥 2 + 23𝑥 + 44 = 0
3x2+12x+11x+44=0
3x(x+4)+11(x+4)=0
(3x+11)(x+4)=0
11
x= -4,- 3
II. 4𝑦 2 + 33𝑦 + 65 = 0
4y2+20y+13y+65 =0
4y(y+5)+13(y+5)=0
(y+5)(4y+13)=0
13
y = -5, -
4
So, No relation

63 www.bankersadda.com | www.sscadda.com | www.careerpower.in | Adda247 App


Daksh Quant Question Bank "June Edition" By Shantanu Shukla
S88. Ans.(d) (𝑥 − 3)(𝑥 − 15) = 0
Sol. I. 2x² + 7x – 60 = 0 𝑥 = 3,15
2x² + 15x – 8x – 60 = 0 II. 𝑦 2 + 12𝑦 − 45 = 0
x(2x + 15) – 4 (2x + 15) = 0 𝑦 2 + 15𝑦 − 3𝑦 − 45 = 0
(x – 4) (2x + 15) = 0 (𝑦 − 3)(𝑦 + 15) = 0
−15
𝑥 = 4, 𝑦 = 3, −15
2
Clearly, 𝑥 ≥ 𝑦
II. 3y² - 28y + 64 = 0
3y² - 12y- 16y + 64 = 0
S293. Ans.(e)
3y (y- 4) – 16 (y – 4) = 0
(3y – 16) (y – 4) = 0 Sol. I. 9𝑥 2 + 11𝑥 + 2 = 0
16 9𝑥 2 + 9𝑥 + 2𝑥 + 2 = 0
𝑦 = ,4 (9𝑥 + 2)(𝑥 + 1) = 0
3
y≥x 2
𝑥 = − 9 , −1
S289. Ans.(e) II. 8𝑦 2 + 6𝑦 + 1 = 0
Sol. I. x² - 2x – 24 = 0 8𝑦 2 + 4𝑦 + 2𝑦 + 1 = 0
x² - 6x + 4x – 24 = 0 (4𝑦 + 1)(2𝑦 + 1) = 0
1 1
x(x – 6) + 4 (x – 6) = 0 𝑦 = − ,−
2 4
(x + 4) (x- 6) = 0 Clearly, no relation can be established
x= 6, -4
II. y² + 3y – 40 = 0 S294. Ans.(c)
y² + 8y – 5y – 40 = 0 Sol. I. 6𝑥² + 5𝑥 + 1 = 0
y(y + 8) – 5 (y + 8) = 0 6𝑥 2 + 3𝑥 + 2𝑥 + 1 = 0
(y – 5) (y + 8) = 0 (3𝑥 + 1)(2𝑥 + 1) = 0
y = 5, -8 𝑥 = − ,−
1 1

No relation can be established 3 2


II. 4𝑦 2 – 15𝑦 = 4
4𝑦 2 − 16𝑦 + 𝑦 − 4 = 0
S290. Ans.(c)
(4𝑦 + 1)(𝑦 − 4) = 0
Sol. I. 4x² + 27x + 45 = 0 1
4x² + 12x + 15x + 45 = 0 𝑦 = − ,4
4
4x(x + 3) + 15 (x + 3) = 0 Clearly, 𝑥 < 𝑦
(4x + 15) (x + 3) = 0
𝑥=
−15
, −3 S295. Ans.(c)
4
Sol. I. 𝑥 2 + 3𝑥 = 0
II. 5y² + 42y + 88 = 0
𝑥(𝑥 + 3) = 0
5y² + 20y + 22y + 88 = 0
𝑥 = 0, −3
5y (y + 4) + 22 (y + 4) = 0
II. 𝑥 2 + 𝑦 = 10
(5y + 22) (y + 4) = 0
−22 𝑦 = 10 − 𝑥 2
𝑦 = −4, 5 𝑖𝑓 𝑥 = 0, 𝑦 = 10
𝑥>𝑦 𝑖𝑓 𝑥 = −3, 𝑦 = 10 − (−3)2 = 1
Clearly, 𝑥 < 𝑦
S291. Ans.(e)
Sol. I. 𝑥 2 + 5𝑥 + 6 = 0 S296. Ans.(c)
𝑥 2 + 3𝑥 + 2𝑥 + 6 = 0 Sol. I. 𝑥 2 − 25𝑥 + 156 = 0
(𝑥 + 3)(𝑥 + 2) = 0 x2 -12x -13x +156=0
𝑥 = −2, −3 x(x-12)-13(x-12)=0
II. 𝑦 2 + 9𝑦 + 14 = 0 (x-12)(x-13)=0
𝑦 2 + 7𝑦 + 2𝑦 + 14 = 0 x =12,13
(𝑦 + 2)(𝑦 + 7) = 0 II. 𝑦 2 − 29𝑦 + 210 = 0
𝑦 = −2, −7 y2-14y -15y +210=0
Clearly, no relation can be established y(y-14) -15(y-14)=0
(y-14)(y-15)=0
S292. Ans.(b) y=14,15
Sol. I. 𝑥 2 − 18𝑥 + 45 = 0 So,x<y
𝑥 2 − 15𝑥 − 3𝑥 + 45 = 0

64 www.bankersadda.com | www.sscadda.com | www.careerpower.in | Adda247 App


Daksh Quant Question Bank "June Edition" By Shantanu Shukla
S297. Ans.(d) When article sold at actual selling price,
Sol. I. 𝑥 2 = 196 𝑥−𝑦
3𝑦
−𝑦
Profit % = × 100 = 2
× 100 = 50%
x =√196 𝑦 𝑦
x =±14
II. 𝑦 = √196 S302. Ans.(e)
y =14 Sol. let CP be Rs. x
130
So,x ≤ y MP = 100 × 𝑥 = 𝑅𝑠. 1.3𝑥
90
SP (given) = 100 × 1.3𝑥 = 𝑅𝑠. 1.17𝑥
S298. Ans.(e) 85
Sol. I. 𝑥 2 + 12𝑥 + 35 = 0 Earlier SP (announced) = 100 × 1.3𝑥 = 𝑅𝑠. 1.105𝑥
x2+5x+7x+35=0 Gain = 1.17𝑥 − 1.105𝑥 = 𝑅𝑠. 0.065𝑥
x(x+5)+7(x+5)=0 0.065𝑥 = 13
(x+5)(x+7)=0 𝑥 = 𝑅𝑠. 200
x= -5,-7
II. 𝑦 2 + 14𝑦 + 48 = 0 S303. Ans.(a)
y2+ 6y+8y+48 =0 Sol. let CP of bags be Rs. 4x & Rs. 5x respectively.
y(y+6)+8(y+6)=0 110 120
Total SP of bags = 100 × 4𝑥 + 100 × 5𝑥 = 4.4𝑥 + 6𝑥 =
(y+8)(y+6)=0
y = -8,-6 𝑅𝑠. 10.4𝑥
10.4𝑥−9𝑥 5
So, no relation. Required Profit % = × 100 = 15 %
9𝑥 9

S299. Ans.(a) S304. Ans.(b)


Sol. I. 3𝑥² + 23𝑥 + 30 = 0 Sol. Let cost price of the item be 100x
3x2 +18x +5x +30 =0 Marked price of the item=100x+ 100x ×
60
=160x
3x(x+6) +5(x+6)=0 100

(3x+5)(x+6)=0 Selling price of items after giving discounts=160x ×


90 85
5 × =122.4x
x= -6,− 3 100 100
122.4𝑥−100𝑥
II. 𝑦² + 15𝑦 + 56 =0 Profit percentage= 100𝑥
× 100 =22.4 %
y2 +8y +7y +56 =0
y(y +8)+7(y+8)=0 S305. Ans.(c)
(y+7)(y+8)=0 Sol. Let original cost price of the article be Rs.100x.
y =-7,-8 110
So, original selling price of the article = 100𝑥 × 100
So, x > y
= Rs.110x
95
S300. Ans.(c) Now, new cost price of the article = 100𝑥 × = Rs.95x
100
Sol. I. 𝑥 2 + 17𝑥 + 72 = 0 And, new selling price of the article = Rs.(110x + 120)
x2+ 8x+9x+72=0 ATQ,
120
x(x+8)+9(x+8)=0 95𝑥 × = 110𝑥 + 120
100
(x+9)(x+8)=0 ⇒ 4𝑥 = 120
x = -8,-9 x = 30
II. 𝑦 2 + 13𝑦 + 42=0 So, cost price of the article = 100x = Rs.3000
y2 +6y+7y+42 =0
y(y+6)+7(y+6)=0 S306. Ans.(c)
(y+6)(y+7)=0 Sol. distance covered is directly proportional to speed
y = -6,-7 When they start at same time, they will cover distance in
So,x<y
ratio of their speeds
Let distance covered by Kappu & Chandu be 5x km & 6x
S301. Ans.(e)
km respectively
Sol. let actual SP be Rs. x 6𝑥−5𝑥
4𝑥 Required answer = 6𝑥+5𝑥 × 110 = 10 𝑘𝑚𝑠
New selling price = Rs. 5
Let CP be Rs. y
4𝑥 S307. Ans.(c)
−𝑦 20 1
ATQ, 5
= = Sol. Let the speed of Abhishek and Rahul be 6x and 5x
𝑦 100 5
4𝑥 𝑦 respectively.
−𝑦=5 6𝑥×5
5
𝑦 2
Required time = 5𝑥 = 6 ℎ𝑜𝑢𝑟𝑠.
𝑥
=3

65 www.bankersadda.com | www.sscadda.com | www.careerpower.in | Adda247 App


Daksh Quant Question Bank "June Edition" By Shantanu Shukla
S308. Ans.(a) S312. Ans.(c)
Sol. let speed of Manoj & Shreya be x & y kmph Sol. Let initial quantity of the mixture in the vessel be x
respectively litre
Let Manoj covers D km in t hours In 20 litre mixture,
𝐷 3
ATQ, 𝑥 = 𝑡 kmph Quantity of alcohol= × 20 = 6 litre
10
2𝐷 4𝐷 7
𝑦= 𝑡 = kmph Quantity of water = 10 × 20 = 14 litre
𝑡 3𝑥
2
−6 1
𝑥 ∶ 𝑦 = 1 ∶ 4 𝑜𝑟 𝑎 ∶ 4𝑎 ATQ, 7𝑥
10
=3
−14+2
Since distance travelled by both will be same (Shreya 3𝑥−60
10
1
catches him) 7𝑥−120
=3
Let time taken by Shreya to cover 20/3 km be k hours 9x -180 =7x -120
𝑥 (𝑘 +
30
) = 𝑦𝑘 x=30 litre
60
𝑎
𝑎𝑘 + 2 = 4𝑎𝑘 S313. Ans.(a)
1
𝑘= ℎ𝑜𝑢𝑟𝑠 = 10 𝑚𝑖𝑛 Sol. Let cost price of the mixture =Rs x per kg
6
20 35 50
Speed of Shreya = 3
× 6 = 40 𝑘𝑚𝑝ℎ x
3 2
S389. Ans.(b) (50 -x) : (x -35) = 3 : 2
Sol. Here, the total distance between P to Q is 594 km 50−𝑥 3
=
𝑥−35 2
Relative Speed=(63+54)km/hr
100 -2x =3x -105
=117 km/hr
5x =205
Distance travelled by Train A in 2 hrs=63× 2 = 126 km
x =41
Remaining distance =594 – 126 Selling price of the mixture when sold at 25% profit = 41
=468 km 125
468 ×
Time required to cover the remaing distance= = 4 hrs 100
117 =Rs 51.25 per kg
Distance travelled by Train B in 4 hr= 54× 4 =216 km
Both train will meet at 216 km distance from Q S314. Ans.(b)
Sol.
S310. Ans.(c)
Sol. when time is same then speed is directly proportional
to distance covered
Let speed of Dhoni, Rohit & Virat be x kmph, y kmph & z
kmph respectively
x : y = 1 : 3 or a : 3a
150
z= × 3 = 4.5𝑎 𝑘𝑚𝑝ℎ
100
𝐷
ATQ, =2
𝑎+4.5𝑎
D = 11a km
𝐷 11𝑎
Required time = 4.5𝑎 = 4.5𝑎 = 2.44 hours
⇒7:4
S311. Ans.(b)
Sol. Let quantity of petrol in the vessel be 30x liters S315. Ans.(a)
25 Sol. If x litres of water is added to the mixture, the ratio of
So, quantity of diesel in the vessel = 30𝑥 × 75
milk and water will be 14:5
= 10x liters 14
7
×64
Now, quantity of kerosene in the vessel 5
=1 8
×64+𝑥
100 8
= (30𝑥 × 50 ) − (30𝑥 + 10𝑥) 14
=
56
5 𝑥+8
= 20x liters 14x+112=280
20𝑥
Required ratio = 10𝑥 14x=168
=2:1 x =12 litres

66 www.bankersadda.com | www.sscadda.com | www.careerpower.in | Adda247 App


Daksh Quant Question Bank "June Edition" By Shantanu Shukla
S316. Ans.(c) S321. Ans.(d)
Sol. Let son’s present age= x years Sol. let each invested Rs P
Then, person’s present age=(x+16) year Let Jaddu invested for X years
2
After 2 yrs, (x+16)+2=2(x+2) ATQ,
𝑃×10×𝑋
= 𝑃 (1 +
10
) −𝑃
100 100
x +18=2x +4 𝑋 21
=
x=14 years 10 100
Hence, son’s age after 8 years =14+8= 22 yrs X = 2.1 years

S317. Ans.(c) S322. Ans.(d)


1 20
Sol. Let present ages of Karan and Arjun be 4x & 3x years Interest earned in 1st half of the year= 30000 × 2 × 100
respectively =Rs 3000
4𝑥 = 3𝑥 + 5 Similarly, during 2 nd half , interest earned = 10 % of
𝑥=5 33000 = Rs 3300
Present age of Karan = 4𝑥 = 20 𝑦𝑒𝑎𝑟𝑠 During 2nd year, interest earned
20
Present age of Arjun = 3𝑥 = 15 𝑦𝑒𝑎𝑟𝑠 =(30000+3000+3300)× 100 = Rs 7260
20
Present age of Mahesh = × 5 = 50 𝑦𝑒𝑎𝑟𝑠 Total interest earned at the end of 2 yrs
2
= 3000 +3300 + 7260 = Rs 13560
Required ratio = (50 − 10) ∶ (20 − 10) ∶ (15 − 10) = 40 ∶
10 ∶ 5 = 8 ∶ 2 ∶ 1
S323. Ans.(a)
Sol. Let the investment in A, B and C be 2x, x and 3x
S318. Ans.(d) respectively.
Sol. Let present age of suman’s son be x yr Cumulative interest rate for A, B and C is
Hence, age of suman=(x+25) yr 25 9
10% × 2, (5 + 5 + 100) %, (3 + 3 + 100) %
𝑥+7 1
According to the question, (𝑥+25)+7 = 2 = 20%, 10.25%, 6.09%
2x+14= x+32 20 10.25 3𝑥×6.09
ATQ, 2𝑥 × 100 + 𝑥 × 100 + 100 = 6852
x =32 -14= 18 yrs 68.52𝑥
⇒ 100 = 6852
S319. Ans.(c) ⇒ x = 10000
Sol. Let present age of shivam and ayush be ‘p’ yrs and ‘q’ So, Total amount invested is 60000 Rs.
yrs respectively
120 S324. Ans.(b)
(p+5)= ×𝑝 Sol. Interest received after 3 yrs is Rs 7560 at simple
100
(p+5) =
6𝑝 interest
5 7560
p = 25 Interest received after 1 yrs on S.I =
3
75
Also,(q-6)=(100)× 𝑞 =Rs 2520
2520
3𝑞 Rate of interest(r) = 16800 × 100
q-6 = 4
=15%
q=24 Interest received on C.I at (r+5)% after 2 yrs
Sum of ages of shivam and ayush, 8 yrs hence 20
=16800[(1+100)2 -1]
= 25+8+24+8 36
=65 yrs =16800(25 -1)
11
= 16800( 25)
S320. Ans.(b) =Rs 7392
Sol. Let present age of Father and his son be 3x and x yrs
respectively S325. Ans.(a)
3𝑥+6 7
=3 Sol. ATQ,
𝑥+6 𝑥×14×3 𝑥×10×3
9x+ 18 =7x+42 100
− 100 = 120
(42−30)𝑥
2x =24 = 120
100
x =12 𝑥 = 𝑅𝑠. 1000
Age of son 3 yrs ago=x -3 =12 -3 =9 yrs Required answer = 5𝑥 = 5 × 1000 = 𝑅𝑠. 5000

67 www.bankersadda.com | www.sscadda.com | www.careerpower.in | Adda247 App


Daksh Quant Question Bank "June Edition" By Shantanu Shukla
S326. Ans.(c) S130. Ans.(d)
Sol. Let total work be 30 units (LCM of 15, 30, 10) Sol. P and Q together can complete
2
rd of the total work in
30 𝑢𝑛𝑖𝑡𝑠 3
Efficiency Arshad = = 2
15 𝑑𝑎𝑦 8 days
30 𝑢𝑛𝑖𝑡𝑠
Sanjay = 30 = 1 𝑑𝑎𝑦
Total work can be completed in 12 days by P and Q
Arshad, Sanjay, Vidya =
30
= 6 units/day working together
5 Let the time taken by Q alone to complete the work be ‘b’
days
S327. Ans.(b) 1 1 1
600
Sol. 1 day wage of 4 men & 3 children = 3 = 𝑅𝑠. 200 + =
30 𝑏 12
1 1 1
Let efficiency of a man & a child be M & C units/day 𝑏
= 12
- 30
respectively 1 5−2
Equating total work, 𝑏
= 60
(4𝑀 + 3𝐶) × 3 = 𝑀 × 15 1 3
𝑏
= 60
𝑀: 𝐶 = 3: 1 (𝑡ℎ𝑖𝑠 𝑖𝑠 𝑎𝑙𝑠𝑜 𝑟𝑎𝑡𝑖𝑜 𝑜𝑓 𝑑𝑎𝑖𝑙𝑦 𝑤𝑎𝑔𝑒)
3 Q alone can complete the total work in 20 days
Daily wage of a man = × 200 = 𝑅𝑠. 40 3
15 Time taken to complete 4th work by Q alone
3
S328. Ans.(b) = × 20 =15 days
4
Sol. Let efficiency of a man & a boy be M & B units/day
respectively
S331. Ans.(d)
5𝐵 × 20 = 10𝑀 × 8
𝑀 5 Sol. side of square = √25 = 5 𝑐𝑚
=
𝐵 4 Since non-parallel sides are equal,
Total work = (4 × 5 + 4 × 4) × 3 = 108 𝑢𝑛𝑖𝑡𝑠
Work done by 4 boys in 3 days = 4 × 4 × 3 = 48 𝑢𝑛𝑖𝑡𝑠
Amount earned by boys for their contribution =
48
108
× 540 = 𝑅𝑠. 240

S329. Ans.(d)
Sol. Let, Abhishek can complete the work alone in ‘x’ days.
100
Then, Satish can complete the work alone in x × 75 Height of trapezium = √52 − 32 = 4 𝑐𝑚
1
=
4x
days Area of trapezium = (𝑏𝑎𝑠𝑒1 + 𝑏𝑎𝑠𝑒2) × ℎ𝑒𝑖𝑔ℎ𝑡
2
3 1
Bhavya can complete the work alone in
4𝑥 1
× days 2
× (4 + 10) × 4 = 28 𝑐𝑚2
3 2
2𝑥
= 3 days
S332. Ans.(e)
ATQ,
3 3 3 Sol. let side of square be x cm
4x
+ 2x = 20 𝑥2 4
1+2 1 =5
⇒ 4x = 20 ⇒ x = 15 10𝑥
𝑥 = 8 𝑐𝑚
Bhavya and Abhishek together can complete the work in
15×10 150 Diagonal of square = √2𝑥 = 8√2 𝑐𝑚
= = 6 days.
15+10 25

S333. Ans.(c)
Sol. Let r and h be radius and height of cylinder
respectively.
Now, r + h = 23 cm
ATQ,
2πr (r + h) = 368π
⇒ r = 8 and h = 15
Now, radius of cone = 8 cm.
ATQ,
πr (l + r) = 200π
⇒ l = 17 cm
1
Volume of cone = 3 π × 8 × 8 × 15
= 320 π cm³

68 www.bankersadda.com | www.sscadda.com | www.careerpower.in | Adda247 App


Daksh Quant Question Bank "June Edition" By Shantanu Shukla
S334. Ans.(d) S339. Ans.(d)
Sol. Let radius of smaller & larger circles be r₁ & r₂ A : B : C
respectively. 7000 × 2 6000 × 2 8500 × 2
2π r₁ = 132 + + +
r₁ = 21 m 9000 × 1 7500 × 1 6500 × 1
2π r₂ = 176 ⇒ r₂ = 28 m. = 46 : 39 47
∴ Required difference 39
B’s profit share = 26400 ×
= π(r22 – r12 ) 132
22 = Rs 7800
= × 49 × 7
7
= 1078 m² S340. Ans.(e)
Sol. Let x = Amount invested by ‘A’ and y = amount
S335. Ans.(b)
invested by ‘B’
Sol. let side of 4 squares be a,b,c & d cm respectively
24 Ratio of profit of A, B & C = (𝑥 × 12): (𝑦 × 9): (12000 × 3)
𝑎 = 4 = 6 𝑐𝑚 = 4𝑥: 3𝑦: 12000
32
𝑏= = 8 𝑐𝑚 ATQ,
4 4𝑥 48
40 = 24 ⇒ 𝑥 = 6,000
𝑐= 4
= 10 𝑐𝑚 12,000
48 3y 48
𝑑 = = 12 𝑐𝑚 and = ⇒ y = 8,000
4 12,000 24
Perimeter of new square = 𝑎 + 𝑏 + 𝑐 + 𝑑 = 6 + 8 + 10 + Required sum = 6,000 + 8,000 = Rs.14,000
12 = 36 𝑐𝑚
4(𝑠𝑖𝑑𝑒) = 36 S341. Ans.(b)
𝑠𝑖𝑑𝑒 = 9 𝑐𝑚 Sol. Let speed of current be x kmph
Required area = 𝑠𝑖𝑑𝑒 2 = 92 = 81 𝑐𝑚2 ATQ,
10.8 36
= 60
S336. Ans.(d) (21−𝑥)

Sol. Let ratio of P’s investment and Q’s investment be x:y ⇒ x = 3 kmph
Therefore, profit will be shared in the ratio 4x:5y Now, downstream speed = 21 + 3 = 24 kmph
60
Given,
4𝑥
× 75000 =15000 Total time taken =
4𝑥+5𝑦 24
4𝑥 1 = 2 hours 30 minutes
=
4𝑥+5𝑦 5
20x =4x +5y S342. Ans.(b)
16x =5y 36
Sol. Downstream speed = 4 = 9 km/hr
y : x=16:5
1
Speed of the current= × 9 = 3 km/hr
3
S337. Ans.(d) Speed of the boat= 9 – 3 = 6 km/hr
Sol. A : B : C Now , Uptream speed = 6 – 3 =3 km/hr
Amount 2500 4500 2400 78
Total time taken = 3 = 26 hr
Time period 12 12 7
Reqd. ratio 25 : 45 : 14
Required difference in profit share of B and C=(45-14)× S343. Ans.(c)
16800 Sol. let speed of stream be x km/hr
84
Speed of boat in still water=4x km/hr
=Rs 6200 220 108
4𝑥+𝑥
+ 4𝑥−𝑥 =20
S338. Ans.(a) 220
+
108
=20
Sol. Ratio of investment of Arun, bhavya & Ashu 5𝑥
44
3𝑥
36
4×3:x×3:4×x 𝑥
+ 𝑥
=20
Ratio of profit 80
=20
𝑥
24 × 12 : 24 × 3x : 24 × 4x
x=4 km/hr
ATQ -
4𝑥 1850 speed of stream= 4 km/hr
7𝑥+12
= 3700 speed of boat in still water= 4x=16 km/hr
8x = 7x + 12 40 48
Reqd . sum=20 + 12 = 2 + 4 = 6 ℎ𝑟𝑠
x = 12

69 www.bankersadda.com | www.sscadda.com | www.careerpower.in | Adda247 App


Daksh Quant Question Bank "June Edition" By Shantanu Shukla
S344. Ans.(e) Expenditure of Manoj = 9000 – (4500 – 500) = Rs 5000
Let speed of stream be u km/hr 100
Average expenditure of Sanjay & Irfan = 90 × 4500 = Rs
According to the question,
54 54 5000
+ =7.5 Expenditure of Irfan = 10000 – 4500 = Rs 5500
15+𝑢 15−𝑢
18 18 5 5000+5500
+ = Required average = = Rs 5250
15+𝑢 15−𝑢 2 2
18(15−𝑢+15+𝑢) 5
=
(15+𝑢)(15−𝑢) 2
216=225-u2 S350. Ans.(e)
u2=9 Sol. required average cost
200+2×80+3×95 645
u=3 km/hr = 8
= 8 = 𝑅𝑠 80.625
48 48
Time required to travel 48 km in upstream= = =4 hrs
15−3 12
S351. Ans.(a)
S345. Ans.(d) Sol. total students in a section = students failed in both +
105
Sol. In still water , the speed of boat = 6 = 17.5 km/hr. students passed in half yearly + students passes in annual
– students passed in both
And let the rate of stream be V km/hr
total students in section B = 15 + 30 + 25 − 20 = 50
According to the question,
𝑉 9
= S352. Ans.(d)
(17.5−𝑉) 26
26V =157.5 -9V Sol. students failed in both exams in all sections
35V =157.5 = 10 + 15 + 20 = 45
V =4.5 km/hr Students passed in both exams in all sections
Total time taken to travel 364 km roundtrip = 20 + 20 + 25 = 65
364 364
=(17.5−4.5) + (17.5+4.5) Required % =
65−45
× 100 = 44 %
4
45 9
364 364
= +
13 22
= 44.54 hrs S353. Ans.(c)
= 45 hrs. (approx.) Sol. students passed in only one examination in all
sections
S346. Ans.(a) = (30 + 40 − 20) + (30 + 25 − 20) + (35 + 30 − 25)
Sol. Expenditure of A = 2400 Rs. = 125
125
Now, 4 → 2400 Required average = = 41.67
3
1 → 600
Average expenditure of A, B and C S354. Ans.(e)
600×(4+2+5)
= 3
= 2200 𝑅𝑠. Sol. Total students in section C = 20 + 35 + 30 − 25 = 60
20
Required % = 60 × 100 = 33.33%
S347. Ans.(d)
Sol. Let no. of questions he attempted correct be x.
S355. Ans.(b)
ATQ, 3x – 0.5 (250 – x) = 435
Sol. students in section A = 10 + 30 + 40 − 20 = 60
3.5x – 125 = 435
Students in section B = 15 + 30 + 25 − 20 = 50
x = 160
Students in section C = 20 + 35 + 30 − 25 = 60
S48. Ans.(d) Section A & C have same no. of students
Sol. Sum of ages of all the 20 members = 20 × 25 = 500
Sum of ages of first 18 members = 18 × 24 = 432 S356. Ans.(c)
Sum of ages of last 2 members = 500 –432 = 68 Sol. Total marks scored by lokesh in physics, chemistry
80 76 84
68
∴ Average age= 2 = 34 and maths together= 150× 100 +150× 100 + 150 × 100
=120+114+126 =360
S349. Ans.(d) Total marks scored by Amit in physics, chemistry and
70 66 58
Sol. let Sanjay spends Rs x. maths together=150× 100 +150× 100 + 150 × 100
Expenditure of Nawaz = x – 500 Rs = 105 + 99 + 87 =291
ATQ, x+x-500=8500 Required difference =360 – 291 =69
X= Rs 4500

70 www.bankersadda.com | www.sscadda.com | www.careerpower.in | Adda247 App


Daksh Quant Question Bank "June Edition" By Shantanu Shukla
S357. Ans.(d) A B C
Sol. Total marks scored by Siddharth in all the Pen 7x=7×33 3y=3×70 128
5z= 8 ×5
48 72 88 70
subjects=150× +150× + 150 × +100× + =231 =210
100 100 100 100 =80
86
100 × 100 Pencil 5x=5×33 2y= 2×70 128
3z= 8 ×3
=72+108+132+70+86 =468 =165 =140
=48
468
overall percentage marks scored by Siddharth= 650 × 100
S361. Ans.(c)
= 72%
Sol. Total amount received by selling all pen by A=231×
20 = Rs 4620
S358. Ans.(a)
Total amount received by selling all pencil by A =165×10
Sol. Total marks scored by Ritesh in all the subjects=150×
76 82 64 72 94 =Rs 1650
+150× + 150 × +100× + 100 × Total amount earned by selling all pen &pencil by A
100 100 100 100 100
=114+123+96+72+94 =499 =4620+1650 =Rs 6270
50
Total marks scored by Aakash in all the subjects=150× 100
64 78 65 75 S362. Ans.(b)
+150× 100 + 150 × 100 +100× 100 + 100 × 100 Sol. Total pens sold by A and B together = 231+210 =441
=75+96+117+65+75 =428 Total pencil sold by B and C together=140 +48 =188
Required difference =499 – 428 =71 441
Required ratio = 188 =441:188
S359. Ans.(c)
S363. Ans.(d)
Sol. marks scored in physics subject by all the given five 231+210+80 521
students together=150×
66
+150×
64
+ 150 ×
72 Required average= 3
= 3 =173.67
100 100 100
76 82
+150× 100 + 150 × 100 S364. Ans.(a)
=99+96+108+114+123 =540 number of pens sold by stationary B after increase of 20
540 120
Average marks scored in physics= 5 =108 %=210× 100 =252
number of pencil sold by stationary C after increase of 25
S360. Ans.(b) %=48×
125
=60
Sol. Total marks scored by Aakash, Siddharth and Lokesh 100
65 70 75 Required sum of pen and pencil =252 +60 =312
in English=100× +100× + 100 ×
100 100 100
=65+70+75 =210 S365. Ans.(c)
Total marks scored by Amit, Aakash and Lokesh in Total pens sold by A ,B and C together =231 +210+80
70 50 80
maths=150× 100 +150× 100 + 150 × 100 = 521
=105+75+120 = 300 Total pencils sold by A ,B and C together =165+140+48
210 = 353
Required percentage =300 × 100 =70% Required difference =521 -353 =168

Solutions (361-365): Let the number of pen and pencil Solutions (366-370): Person who eat only vanilla
sold by A be 7x and 5x respectively and that of by B be 3y = 100- (40+10+30)=20
and 2y respectively. Person who eat butterscotch and chocolate only
Total numbers of pen and pencil sold by A and B = 130-(40+40+30)=20
=7x+5x+3y+2y Person who eat only chocolate
12𝑥 + 5𝑦 =874-128 = 210-(40+40+30+10+20+20) =50
12𝑥 + 5𝑦 =746 Person who eat chocolate= 50+20+30+10= 110
Now,
110
7x =3y ×
100
33𝑦
x = 70
12𝑥 + 5𝑦 =746
33𝑦
12 × 70 +5y =746
396y +350y =746 × 70
746×70
y = 746 = 70
33𝑦 33×70
x = 70 = 70
=33

71 www.bankersadda.com | www.sscadda.com | www.careerpower.in | Adda247 App


Daksh Quant Question Bank "June Edition" By Shantanu Shukla
S366. Ans.(a) S376. Ans.(c)
Sol. Number of people who eat only chocolate=50 Sol. amount received by Rohit

S367. Ans.(a) = 4000 +


4000×10×2
= 𝑅𝑠. 4800
100
Sol. A.T.Q
People eating chocolate and butterscotch only = 20
S377. Ans.(e)
People eating only butterscotch =40
20 Sol. interest amount received by Karan
∴ required percentage = 40 × 100 = 50% 8000×10×2
= = 𝑅𝑠. 1600
100
Interest amount received by Mahesh
S368. Ans.(d) 6000×12×4
Sol. people eating only vanilla = 20 = 100
= 𝑅𝑠. 2880
People eating all 3 icecreams = 30 Required % =
2880−1600
× 100 = 80%
1600
Required difference = 30- 20= 10
S378. Ans.(d)
S369. Ans.(c)
Sol. total interest amount received by Anurag & Rohit
Sol. people eating chocolate= 110 4000×16×4 4000×10×2
People eating vanilla= 100 together = 100
+ 100
= 𝑅𝑠. 3360
110
∴ 𝑟𝑒𝑞𝑢𝑖𝑟𝑒𝑑 𝑝𝑒𝑟𝑐𝑒𝑛𝑡𝑎𝑔𝑒 = × 100 = 110 %
100
S379. Ans.(a)
8000×10×2
S370. Ans.(b) Sol. interest received by Karan (SI) = 100
Sol. people eating only chocolate and only butterscotch = 𝑅𝑠. 1600
together= 50+40= 90 Interest received by Karan (CI)
People eating only vanilla = 20 10 2
= 8000 (1 + 100) − 8000 = 𝑅𝑠. 1680
∴ 𝑟𝑒𝑞𝑢𝑖𝑟𝑒𝑑 𝑟𝑎𝑡𝑖𝑜 = 9: 2
Required value = 1680 − 1600 = 𝑅𝑠. 80
S371. Ans.(d)
Sol. required difference = average marks scored by S380. Ans.(e)
8000×10×2
Student A - Average marks scored by Student B Sol. Interest received by Karan = = 𝑅𝑠. 1600
100
70+90+60+55 50+80+75+65 5
∴ 4
− 4
= 4 = 1.25 Interest received by Anurag =
4000×16×4
= 𝑅𝑠. 2560
100
6000×12×4
Interest received by Mahesh = = 𝑅𝑠. 2880
S372. Ans.(c) 100
4000×10×2
Sol. marks obtained by student A in Math and Computer Interest received by Rohit = 100
= 𝑅𝑠. 800
together =70 + 90 = 160 Clearly, Mahesh had received highest interest
marks obtained by student B in Science and English
together=75+65 =140 S381. Ans.(d)
required ratio = 160:140= 8:7 Sol. let his total expenditure be Rs. x in July
40 1 𝑥
Savings = 100 × 𝑥 × 2 = 𝑅𝑠. 5
S373. Ans.(b) 𝑥
Sol. Overall percentage marks of Student B = ATQ, 𝑥 + 5 = 12000
50+80+75+65 𝑥 = 𝑅𝑠. 10000
× 100 = 67.5
400 30 30
Expenditure on food = 100 𝑥 = 100 × 10000 = 𝑅𝑠. 3000
S374. Ans.(c)
Sol. Marks Scored by Student A in Math =70 S382. Ans.(a)
Marks Scored by Student B in Science and English Sol. let salary & savings be Rs. x & Rs. y respectively for
=75+65=140 March & June
70
Required % = 140 × 100 =50% Expenditure in March = expenditure in June = 𝑅𝑠. (𝑥 − 𝑦)
35
Expenditure on travel in March = 𝑅𝑠. × (𝑥 − 𝑦)
100
S375. Ans.(b) 40
Expenditure on food in June = 𝑅𝑠. 100 × (𝑥 − 𝑦)
40
Sol. A.T.Q, passing marks = × 120 = 48 35
100 Required % = 40 × 100 = 87.5%
∴ required difference = 80 – 48 = 32

72 www.bankersadda.com | www.sscadda.com | www.careerpower.in | Adda247 App


Daksh Quant Question Bank "June Edition" By Shantanu Shukla
S383. Ans.(e) Let winning candidate got x% of votes cast and Losing
Sol. let total expenditure in May & July is Rs. 5x & Rs. 4x Candidate got (x-12)% of votes cast.
respectively. Now, ATQ
35 30 𝑥 + 𝑥 − 12 = 100
Required ratio = (100) × 5𝑥: (100) × 4𝑥 = 35 ∶ 24
𝑥 = 56%
44
S384. Ans.(c) Votes obtained by losing candidate = 100 × 1800 = 792
90
Sol. expenditure in March = 100 × 5000 = 𝑅𝑠. 4500
S393. Ans.(e)
40
Expenditure on rent in March = 100 × 4500 = 𝑅𝑠. 1800 Sol. average registered voters of B,C,D
(25+20+15) 10000
90
Expenditure in July = 100 × 8000 = 𝑅𝑠. 7200 = 100
× 3 = 2000
40
Expenditure on rent in July = × 7200 = 𝑅𝑠. 2880
1800+2880
100 S394. Ans.(c)
Required average = 2
= 𝑅𝑠. 2340 Sol. votes cast -
20 70
A = 10000 × 100 × 100 = 1400
S385. Ans.(c) 25 65
Sol. let equal expenditure be Rs. x. B = 10000 × 100 × 100 = 1625
35 30 15 80
𝑥− 𝑥 5 D = 10000 × 100 × 100 = 1200
Required % = 100
30
100
× 100 = × 100 = 16.67% 20 75
𝑥 30
100 E = 10000 × 100 × 100 = 1500
Maximum voters cast their votes in village B.
S386. Ans.(c)
Sol. total Samsung mobiles
S395. Ans.(b)
= 2400 + 4400 + 1800 + 2800 = 11400
Sol. average number of registered voters from village A &
10000 20+20
S387. Ans.(e) C = 2 × 100 = 2000
Sol. required answer Average no. of registered voters from village B, D & E
= (2300 + 2500) − (1800 + 2800) = 200 =
10000
×
(25+15+20)
= 2000
3 100
2000
S388. Ans.(d) Required % = 2000 × 100 = 100%
1800 2
Sol. required % = 2700 × 100 = 66 3 %
S396. Ans.(c)
Sol. Total number of males employees in company E
S389. Ans.(a) 22 2
Sol. required ratio = 5400× × =792
100 3
= (2300 + 2500 + 3500) ∶ (2400 + 4400 + 2800) Total number of female employees in company D
20 3
= 83: 96 = 5400× 100 × 5 =648
792
S390. Ans.(e) Required ratio= =11 : 9
648
2500−2300
Sol. Nokia (2017) = 2300 × 100 = 8.7%
3500−2500
S397. Ans.(a)
Nokia (2018) = × 100 = 40% Sol. Total number of male employees in company
2500
2800−1800 18 2
Samsung (2019) = × 100 = 55.55% A=5400× × =648
1800 100 3
Nokia (2019) =
2700−3500
× 100 = 23% (decrease) Total number of female employees in company E
3500 22 1
4400−2400 = 5400× × =396
Samsung (2017) = 2400 × 100 = 83.33% 100 3
648
Clearly, Samsung in 2017 shows maximum production Required percentage=396 × 100 = 163.63 %
increase =164% (approx.)

S391. Ans.(a) S398. Ans.(b)


Sol. no. of valid votes cast in village B Sol. total male employees in company B,C and D together
25 80 90 28 3 12 1 20 2
= 10000 × 100 × 100 × 100 = 1800 =5400× × +5400× × + 5400× ×
100 4 100 3 100 5
= 1134+216+432
S392. Ans.(d) =1782
1782
Sol. total valid votes cast in village C Required percentage=5400 × 100 =33%
20 90
= 10000 × 100 × 100 = 1800

73 www.bankersadda.com | www.sscadda.com | www.careerpower.in | Adda247 App


Daksh Quant Question Bank "June Edition" By Shantanu Shukla
S399. Ans.(d)
Sol. Total female employees in all the 5 companies
together
18 1 28 1 12 2
=5400× 100 × 3 +5400× 100 × 4 + 5400× 100 × 3 +
20 3 22 1
5400× 100 × 5 +5400× 100 × 3
=324+378+432+648+396
=2178

S400. Ans.(e)
Sol. Central angle of total employees from company B and
360
D together=(28+20)× 100 =172.8°

74 www.bankersadda.com | www.sscadda.com | www.careerpower.in | Adda247 App

You might also like